Сохранен 508
https://2ch.hk/spc/res/108409.html
Домены arhivach.top и arhivach.site временно не функционируют! Используйте домен ARHIVACH.XYZ.
24 декабря Архивач восстановлен после серьёзной аварии. К сожалению, значительная часть сохранённых изображений и видео была потеряна. Подробности случившегося. Мы призываем всех неравнодушных помочь нам с восстановлением утраченного контента!

Тред тупых вопросов и ответов. 13

 Аноним Вск 06 Апр 2014 19:22:16 #1 №108409 
1396797736654.jpg

И снова с вами — лучший, после огурцов, тред спейсача! Тред гениальных вопросов о вот этих вот наших галактиках со вселенными.

Спрашиваем то, за что в других местах выдают путёвку в биореактор. Здесь анонимные ученые мирового уровня и просто любители критически рассмотрят любые гениальные идеи и нарисованные в Paint схемы, ответят на смелые рассуждения будущих светил космонавтики, вступят друг с другом в околонаучный диспут.

Прошлый: http://2ch.hk/spc/res/105995.html

Аноним Вск 06 Апр 2014 19:25:56 #2 №108411 

>>108409
Почему религиозные американцы не крестят свои ракеты?

Аноним Вск 06 Апр 2014 20:33:12 #3 №108417 

Ебанный стыд...
Во-первых, Алькубьерре.
Во-вторых, не упасть, а создавать вокруг корабля изнутри (иначе кина не будет).
В-третьих, НАСА искривляет пространство на десятимиллионную часть, контролируя это сверхточными интерферометрами, до самого варп-привода здесь - как до Антарктиды раком.

Аноним Вск 06 Апр 2014 21:44:57 #4 №108426 
1396806297823.png

>>108411
Они считают ракеты бездушными механизмами.

Аноним Вск 06 Апр 2014 22:13:46 #5 №108434 
1396808026926.png

>>108426
Тогда почему попы не выглядят так?

Аноним Вск 06 Апр 2014 22:17:06 #6 №108435 

>>108434
Дикари же.

Аноним Вск 06 Апр 2014 22:46:26 #7 №108443 

>>108411
Потому что они в основном протестанты, а у них не принято освящение чего-либо. Что не мешало экипажу Аполлона-8 в прямом эфире с орбиты Луны читать Библию.

Аноним Вск 06 Апр 2014 23:46:01 #8 №108445 

Аноны, помогите вспомнить. Когда я был малым, то в детском лагере в библиотеке взял книжку про космос. Так вот книжка советская, конца 80-х, суть ее в проектах советской космонавтики, т.е. межзведных путешествиях, станциях и т.п.
Было очень много красочных иллюстраций. Буду благодарен если кто читал ее и поможет мне ее найти.

Аноним Пнд 07 Апр 2014 07:20:55 #9 №108460 

>>108275
Ну а как ты себе представляешь игру в такой вселенной? Про что она будет? Что ты хочешь там увидеть отличного от той же eve?
Под твой запрос больше походит какой-нибудь рассказ. Или та же шкала времени, что ты давал. То есть отыгрыш в голове, на основе подробных описаний объектов сможешь представлять всё это.

Аноним Пнд 07 Апр 2014 16:43:50 #10 №108483 
1396874630067.jpg

>>108417
Пожалуйста, исправьте в пасте слово "ебаный", выпали глазки.

Аноним Пнд 07 Апр 2014 19:46:10 #11 №108498 

>>108460
>Ну а как ты себе представляешь игру в такой вселенной?
Космосим типа илиты с возможностью гулять по поверхностям планет и искать в разрушенных человеческих городах причину исчезновения человечества
>Про что она будет?
Нас отправили в стазисе в ПУЗЫРЕ АЛЬКУБЕРКЕ к краю Вселенной но внезапно что то дало сбой и мы оказались в 100+ трлн году от рождения чизаса. Фриплей, можно бродить по руинам, копать астероиды, летать, изучать объекты космоса(и открывать новые технологии благодаря этому. Начинаем с ТЯРД/ЯРД, ПУКЗЫРЁМ АЛЬУКУБЕРКЕ, кораблём из карбона(графен, нанотрубки, етц) на 1-5с, под конец имеем йоба генератор на чёрных дырах(и двигателем на его основе, кораблём из вещества НЗ). Твёрдый сай фай. Цель игры - узнать о том куда делось человечество
>Что ты хочешь там увидеть отличного от той же eve?
Не симулятор экселя, твёрдый сай фай без жидкого вакуума

Аноним Пнд 07 Апр 2014 19:54:21 #12 №108500 

>>108498
>триллионы лет после
>в разрушенных человеческих городах
Дальше не читал, уже очевидно что ты дебил.
А жаль.

Аноним Пнд 07 Апр 2014 20:25:49 #13 №108505 

>>108500 Наномашины чинящие города + йоба материталы. Уже очевидно что ты дебил думающий в рамках современных технологий

Аноним Пнд 07 Апр 2014 20:32:03 #14 №108506 

>>108505
> >разрушенных человеческих городах
> >бродить по руинам
>Наномашины чинящие города
Больше оправданий, ребенок. А лучше просто сьеби в /сф - там как раз резервация для инфантильных дурачков.

Аноним Пнд 07 Апр 2014 22:17:53 #15 №108512 

>>108506
>А лучше просто сьеби в /сф - там как раз резервация для инфантильных дурачков
Или что? В сф кстати нет кефирщиков и прочих альтернативно одарённых И мне как то похуй на твоё мнение. Я просто расписал то что хочу увидеть

Аноним Пнд 07 Апр 2014 22:52:25 #16 №108516 

>>108512
>В сф кстати нет кефирщиков
Maybe.
>и прочих альтернативно одарённых
Wrong.
Одних только вахадебилов более чем достаточно для полного кефира.
>Я просто
Вижу что просто.
Впрочем, было глупо ожидать чего-то нового или интересного.
Сама идея мне нравится. Проблема в том, что тут две несовместимые идеи - триллионы лет и руины/города. Но раздельно они хороши, да.
А Головачева больше не читай - это слишком треш для твоей слабой психики, няша.

Аноним Пнд 07 Апр 2014 23:04:17 #17 №108517 

>>108516
>Головачева
Кто это такой? С чего ты взял что я его читал?
>триллионы лет и руины/города
Если считать игр условностями + йоба технологии?

Аноним Пнд 07 Апр 2014 23:10:47 #18 №108520 

>>108517
>Кто это такой?
Один колхозный мудак с гигантоманией. Тоже любит чтобы триллионы, и пишет как косноязыкий совок, хоть и с фантазией.
>игр условностями
То есть сразу исходит из того, что говно налеплено. Здравый подход, далеко пойдешь.
>йоба технологии?
Нет. Эта твоя нана за такое время или сдохнет или эволюционирует и сдохнет.
Опять же, не надо смешивать мороженное с черной икрой - получится понятно что.

Аноним Втр 08 Апр 2014 02:36:35 #19 №108545 

>>108498
>твёрдый сай фай
>Начинаем с ТЯРД/ЯРД, ПУКЗЫРЁМ АЛЬУКУБЕРКЕ, кораблём из карбона(графен, нанотрубки, етц) на 1-5с, под конец имеем йоба генератор на чёрных дырах(и двигателем на его основе, кораблём из вещества НЗ)
>оказались в 100+ трлн году от рождения чизаса.
>1-5с
>твёрдый сай фай
Пиздец нахуй.

Аноним Втр 08 Апр 2014 04:30:02 #20 №108555 

Дайте мне ютуб ссулку на передачу канала культуры про исследование проведенное группой ученых, про то , что нашли в реликтовом излучении следы черных дыр прошлой вселенной. В общем эта ссылка была в бреде пару дней назад, посмотреть не было времени, а ссылку сохранить забыл, та передача длится около 40-50 минут. Всем добра и заранее спасибо.

Аноним Втр 08 Апр 2014 09:38:05 #21 №108557 

>>108498
Сделай сам. Не забудь корованы.
>куда делось человечество
Что есть человечество? А через сто лет? А через тысячу? А через всего лишь какой-то миллиард лет здесь уже будет припекать. А через каких-то 100 млрд звездам размером с солнце уже неоткуда будет взяться, все будет засрано железом. Только красно-коричневые карлики. Только хардкор. Ну белые еще.
>человеческих городах
да ты, я смотрю, мамкин планетарный шовинист

"Город и звезды", ога

Аноним Втр 08 Апр 2014 12:17:47 #22 №108559 

>>108555
У канала Культура есть сайт с архивом. http://tvkultura.ru/video/show/brand_id/28975

Аноним Втр 08 Апр 2014 16:16:14 #23 №108573 

>>108545 что не так? Это вполне возможно запилить и ирл такое может быть. Значит твёрдый. Это же не ГИПЕРПРОСТАРНСТВО, СИЛОВЫЕ ЩИТЫ и етц
>>108520 Не читаю современных рузке писак вообще.
>>108557
>да ты, я смотрю, мамкин планетарный шовинист
Приебался к мелочам, лол. Города могут быть и на орбите. Различные Стэнфордские торы и тому подобное

Аноним Втр 08 Апр 2014 16:24:13 #24 №108575 

>>108573
>к мелочам
Относись к ним серьезней, ибо в мелочах я.

Аноним Втр 08 Апр 2014 19:58:55 #25 №108595 

>>108559
Спасибо! Это то, что я искал!

Аноним Втр 08 Апр 2014 23:09:50 #26 №108605 

Игру он захотел. Будет вам Существование. Будете летать и выяснять, почему другие расы зверушек такие дегенераты и остались на тех уровне средневековья. Можно оставлять после себя руины, ня~.

Аноним Срд 09 Апр 2014 23:10:51 #27 №108726 
1397070651754.jpg

Как мы знаем, что при затмении Луна и Солнце имеют абсолютно идентичные угловые размеры. Как это могло повлиять на нашу планету? Допустим вы взяли две гипотетические копии Земли - З1 и З2, все три полностью одинаковы, за одним исключением - на З1 Луна находится в два раза ближе к планете, а на З3 - в два раза дальше. Как повлияет такое расположение Лун на эти две планеты?

sageАноним Чтв 10 Апр 2014 01:56:03 #28 №108748 

>>108726
Навскидку - при "близкой" луне сильнее приливные силы => выше приливы и сильнее тектоника коры планеты.

Аноним Чтв 10 Апр 2014 13:02:42 #29 №108797 

>>108726
Спешу напомнить, что раньше Луна была ближе, а геологически скоро будет значительно дальше. Но логично, что ключевыми будут изменения да, именно приливных сил.

Аноним Чтв 10 Апр 2014 22:37:41 #30 №108840 

Посоветуйте книг про астрономию. Атласов там, етц. Я сам в этом не шарю. Хочу подарок сделать. Знаю, что человек угорает по астрономии и Хокингу.

Аноним Птн 11 Апр 2014 04:08:34 #31 №108860 

>>108840
Есть подозрение, что интересует твоего человека не столько сама астрономия, сколько раздел астрофизики. Ибо астрономия в общем виде представляет собой созвездия, плоскости эклиптики и прочие звездные координаты, то есть прикладное приложение наблюдений за звездами. Астрофизика же, как подраздел астрономии, как раз изучает черные дыры, темные материи и все эти научно-популярные йоба-объекты в космосе. В общем подари ему "Элегантную вселенную" Грина, если он эту книгу уже читал, то врятли в обычном книжном ты найдешь что-то, что его заинтересует.

Аноним Птн 11 Апр 2014 07:57:59 #32 №108864 

>>108498
> разрушенных человеческих городах
> в 100+ трлн году
Если бы люди так строили...

> Фриплей, можно бродить по руинам, копать астероиды, летать, изучать объекты космоса(и открывать новые технологии благодаря этому. Начинаем с ТЯРД/ЯРД, ПУКЗЫРЁМ АЛЬУКУБЕРКЕ, кораблём из карбона(графен, нанотрубки, етц) на 1-5с, под конец имеем йоба генератор на чёрных дырах(и двигателем на его основе, кораблём из вещества НЗ). Твёрдый сай фай. Цель игры - узнать о том куда делось человечество
По всем параметрам - аутизм в вакууме. Не как что-то плохое.
Но игра слишком специфична. Ближайший аналог - одиночный минесрафт не последней версии (до деревень) с модом типа индустриала и модом, добавляющим заброшенные постройки. Ты один в бесконечном мире, постепенно доходишь до йобаэнергетики и квантоброни. Единственное отличие - не в космосе.
Из существующего по атмосфере подходит noctis: ты один на инопланетном корабле, в галактике, судя по всему, больше никого нет, раса, построившая корабль, куда-то съебала, миллионы звёзд и планет, на некоторых есть руины. Но это больше похоже на space engine, никакой добычи, никакого крафта или развития.

Аноним Птн 11 Апр 2014 13:25:38 #33 №108879 

Тупой вопрос от анона из /wm/:
Почему пилят специальную ракету и одноразовый корабль для полета на Луну, а не выводят Союзами\Протонами блоки, из которых на орбите собирается корабль? И почему нельзя потом, уже улетев с Луны, вывести этот корабль на орбиту, пристыковав скажем к МКС, где он будет дожидаться следующего экипажа и заправки?

Аноним Птн 11 Апр 2014 15:48:13 #34 №108892 
1397216893817.jpg

>>108879
Такие проекты неоднократно разрабатывались, в частности так собирались летать на Луну американцы. В приципе ничего невозможного в этом нет (та же МКС так и построена), но посчитали и предпочли однопусковую схему. Может, на Марс полетят многопусковой.
По второму вопросу - опять же ничего невозможного, но это совершенно ненужная операция - легче сделать два одноразовых корабля, чем один многоразовый. Да и не было во времена полетов на Луну долговременно обитаемых станций. Хотя проекты космических маршруток уже есть, но это в основном для беспилотных частей - с ними спешить некуда, и надежности такой лютой не надо.

Аноним Птн 11 Апр 2014 15:49:59 #35 №108893 

>>108879
>Почему пилят специальную ракету
Потому что нужна очень большая ракета.
>одноразовый корабль для полета на Луну
Если ты про ППТС, то он будет частично многоразовым.
>И почему нельзя потом, уже улетев с Луны, вывести этот корабль на орбиту
Зачем? Алконавты на жопе будут домой спускаться?

Аноним Птн 11 Апр 2014 15:53:43 #36 №108894 

Правильно ли я понимаю, что все материальные объекты притягиваются друг к другу. Например я к телефону или он ко мне. Или же только легкое к тяжелому?

Аноним Птн 11 Апр 2014 16:11:51 #37 №108897 

>>108894
Все со всем имеет гравитационное взаимодействие, но оно настолько слабое, что его можно заметить только у массивных тел. Ты притягиваешься к телефону и он к тебе, оба вы к Земле, Луне и Солнцу и так далее. Но Солнце и Луна очень далеко, поэтому для тебя заметно только земное притяжение.

Аноним Птн 11 Апр 2014 17:40:22 #38 №108909 

>>108894
Самое забавное, что ты притягиваешь телефон с той же силой, что и он тебя, лол.
Да-да, и Землю ты притягиваешь с той же силой, что она тебя. Просто Земля большая, а ты маленький дурашка, поэтому разница в ускорениях, которые вы друг другу сообщаете, такая, что замечаешь ты только притяжение Земли.

Аноним Птн 11 Апр 2014 22:08:28 #39 №108939 

>>108909
Наркоман штоле?

Аноним Птн 11 Апр 2014 22:45:03 #40 №108940 

>>108939
Нет, просто ты еще слишком маленький, что бы понять третий закон Ньютона. Ну, ничего, через несколько лет учительница будет тебе рассказывать подробно.

Аноним Птн 11 Апр 2014 22:55:35 #41 №108941 

Тред не читай - тупой вопрос задавай!

При взрыве гиперновой ее ядро превращается в черную дыру. Это значит, что у черной дыры есть какой-то свой диаметр. Что будет, если за десятки миллиардов лет эта дыра просто-напросто заполнится до отказа поглощенным веществом? И превратится в шар абсолютно нового металла? Может такое быть?

Аноним Птн 11 Апр 2014 23:30:51 #42 №108947 

>>108941
ни во что, кроме железа, она превратиться не может

Аноним Птн 11 Апр 2014 23:32:39 #43 №108948 

>>108947
Хорошо. Но есть ли вероятность наполнения черной дыры?

Аноним Суб 12 Апр 2014 00:34:06 #44 №108953 

>>108941
>>108948
Размер ЧД увеличивается вместе с ее массой (r=2GM/c^2). Так что, не бойся, не переполнится: Б-г впилил динамическое выделение ресурсов Вселенной.
А некая абстрактная "средняя плотность" ЧД (тупо масса/объем) с ростом массы падает. У ЧД с массой Солнца плотность уровня ядерной, то у сверхтяжелых ЧД - меньше плотности воды. Но вся эта плотность абстрактна, и судить о свойствах подгоризонтной материи по ней нельзя - нужно учитывать существование сингулярности.
Спонсор ликбеза - Загнивающая.

Аноним Суб 12 Апр 2014 00:44:16 #45 №108955 

>>108953
А может ли она каким-нибудь образом распасться или как-то иначе умереть?

Аноним Суб 12 Апр 2014 01:02:15 #46 №108956 

>>108953
Если свет не может вырваться из-под горизонта событий, то как оттуда вырывается гравитация? Гравитация быстрее света?

Аноним Суб 12 Апр 2014 01:25:22 #47 №108960 

>>108955
Может исхудать до смерти из-за излучения Хокинга. Есть такая мысль, но излучения пока никто не видел, ясен ясень, уж больно оно слабое, если вообще существует.

sageАноним Суб 12 Апр 2014 02:17:52 #48 №108962 

>>108417
Я нихуя не понял. Кто скопипастил мой прошлогодний пост в этот ИТТ тред?

sageАноним Суб 12 Апр 2014 02:53:16 #49 №108963 

>>108956
Эмм... ты сам понял, что сказал? Гравитация не даст распространяться самой себе, лол? По ОТО гравитация вообще - искривление пространства-времени.

Аноним Суб 12 Апр 2014 02:54:21 #50 №108964 

>>108963
сажу забыл

Аноним Суб 12 Апр 2014 03:26:14 #51 №108965 

>>108955
По идее, ее может разорвать приливными силами при взаимодействии с еще более мощной гравитацией.

Аноним Суб 12 Апр 2014 04:02:36 #52 №108967 

>>108445
"Космическая Одиссея"?
>>108265
>http://ru.wikipedia.org/wiki/Временная_шкала_далёкого_будущего
Мда, не стоило читать это ночью, слушая мрачнющий фьюнерал дум метал...
Но в конце есть интересная фишка про возврат Пуанкаре. Лично мне все это кажется куда более эпичным и значимым, чем религиозные сказки народов мира, лол.
>>108156
Очевидно же, что таких игр, может, половина штуки, да и та инди.
А вот в литературе такой сеттинг встречается.
У того же упомянутого Головачева я не говноед, в сосничестве случайно прочитал! в Черном Человеке 2 крайне удачный эпизод про чувака, попавшего в сверхдалекое будущее, когда Солнце остыло, галактики разлетелись, вокруг мертвой Земли (или Солнца, не помню) вращается давным-давно заброшенная станция, не людьми даже построенная, а существами через несколько генераций разумной жизни после гибели человечества. Короче, неожиданно сильно получилось у него.
Еще у Грега Бира "Город в конце времен", как раз 10 триллионов лет от нас, вроде. Но это вообще головоломка, в натуре, легкого чтива не жди.
Может, кто еще вспомнит?

sageАноним Суб 12 Апр 2014 05:16:28 #53 №108969 

>>108962
Да он каждый тред всплывает, считай.

Аноним Суб 12 Апр 2014 08:41:39 #54 №108973 

>>108963
>искривление пространства-времени.
Гравитационное взаимодействие передается со скоростью света, его это смущает.

Аноним Суб 12 Апр 2014 08:53:41 #55 №108975 

>>108963
Да. Раз пространство искривляется, то на горизонте оно скривлено таким образом что любое воздействие не способно его преодолеть. А гравитация как раз воздействие. Значит дыра схлопывается и выпадает из пространства оставляя дырку в которой зияет сингулярность.

Аноним Суб 12 Апр 2014 12:08:24 #56 №108995 

>>108973
>Гравитационное взаимодействие передается со скоростью света
А это разве так? А я думал, оно мгновенно.
Мимо

Аноним Суб 12 Апр 2014 12:21:13 #57 №108996 

>>108967
> Но в конце есть интересная фишка про возврат Пуанкаре
Никакая не интересная, а бред сивой кобылы. Давно обсуждено было, что ко Вселенной такая гипотеза неприменима. ИСЧХ в православной английской вики она и не упоминается

Аноним Суб 12 Апр 2014 12:48:13 #58 №108998 

>>108973
Нихуя у вас тут ЭКРАНИРОВАНИЕ МАССЫ ГРАВИТАЦИЕЙ, блджад!
>>108996
Ясно. Т.е., можно уже начинать перепиливать руку изношенной пилочкой для ногтей?

Аноним Суб 12 Апр 2014 17:10:11 #59 №109020 

>>108998
Успокойся, ты не доживёшь вообще ни до какого интересного асрономического события.

Аноним Суб 12 Апр 2014 17:48:06 #60 №109028 
1397310486962.jpg

Космоанон, а можно ли (и целесообразно ли) применять койлган в качестве ПРО или элемента противокосмической обороны? Приснился сегодня сон, будто Звезду Смерти, которую вывели северокорейцы, японцы расстреляли из электромагнитного ускорителя маленькими шариками, запущенными со скоростью в десятки км/с, словно из какой-то зенитки.
Сам спросил - сам ответил: слишком маленькие болванки нельзя, сгорят в атмосфере. Ну а как-то модернизировать эту идею до рабочего состояния можно?

Аноним Суб 12 Апр 2014 18:01:02 #61 №109030 
1397311262026.jpg

Космонавты на МКС чувствуют перегрузки, когда меняют орбиту станции?

В космосе вообще можно чувствовать перегрузки и на каких скоростях? Допустим это космический истребитель, он сможет остро срезать свою траекторию и крутить петли на огромных скоростях, или нет?

Аноним Суб 12 Апр 2014 18:04:12 #62 №109031 

>>109030
Имеют значения не скорости же, а ускорения.

Аноним Суб 12 Апр 2014 18:04:49 #63 №109032 

>>109030
Ой бля ебать ты лол. Конечно чувствуют, инерцию то никто не отменял. Космический истребитель без гравикомпенсатора ( или инерционного негатора) так не сможет, особенно пилотируемый.Развалится нахуй при резкой смене курса.

Аноним Суб 12 Апр 2014 18:20:31 #64 №109036 

>>109031
>>109032
А на каких скоростях космонавт в невесомости будет чувствовать эквивалент перегрузки в 1 G? Этот параметр как-то связанно с ускорением падения на Земле?

Аноним Суб 12 Апр 2014 18:23:19 #65 №109037 

Котаны, а у меня тут вопрос. Я вот посчитал, что даже с 0.1 световой можно нормально летать до Марса и прочих планет солнечной системы. А как в реальности с двигателями дела обстоят? Какие нибудь разработки есть?

Аноним Суб 12 Апр 2014 18:27:03 #66 №109038 

>>109028
Вот тебе космический шотган.
Нужно разместить на НОО начиненные шариками кубиками черные радионезаметные бомбы. Корректировать орбиты бомб электрическими двигателями. Между сеансами коррекции бомбы не видно. Можно впилить в них ГСН, что бы бомбы по команде могли скучковаться вокруг врага.
Взрыв создает направленный конус траекторий шариков (и отлетающее в противоположном направлении ДНИЩЕ бомбы). Главное, направлять взрыв так, что бы шарики вскоре сгорали в атмосфере.

Или ракету космос-космос. От обычной отличается отсутствием обшивки. Или имеет обшивку "стелс", как понадобится. Еще имеет юбку, которая скрывает от цели выхлоп двигателя.

Аноним Суб 12 Апр 2014 18:27:51 #67 №109039 

>>109030
> Космонавты на МКС чувствуют перегрузки, когда меняют орбиту станции?
В смысле при коррекции орбиты?

Поскольку станция весит дохуя, и приращение скорости выражается в единицах м/с за минуты, то при коррекциях очень малое ускорение, сравнимое с условиями микрогравитации. Максимум у тебя что-нибудь свободно плавающее плавно отлетит к стенке. Где-то даже видео было. Так что нет.

Аноним Суб 12 Апр 2014 18:29:08 #68 №109040 

А что будет, если в гиродин МКС на полном ходу бросить лом?

Аноним Суб 12 Апр 2014 18:30:19 #69 №109041 

>>109037
> А как в реальности с двигателями дела обстоят? Какие нибудь разработки есть?
http://go2starss.narod.ru

Аноним Суб 12 Апр 2014 18:32:22 #70 №109042 
1397313142671.jpg

>>109040
Урановый?

Аноним Суб 12 Апр 2014 18:34:09 #71 №109043 

>>109037
С 0.1с можно будет между звезд летать. Ты ее набери сначала.

> А как в реальности с двигателями дела обстоят? Какие нибудь разработки есть?
Никак не обстоят. Нет.

>>109036
Скорость неважна, важно ускорение. 1g перегрузки будет при ускорении в 2g, т.е. 19.6м/с². Если ты имел в виду чтобы было "как на земле", то нужно ускорение в 1g, т.е. 9.81м/с²

> Этот параметр как-то связанно с ускорением падения на Земле?
Естественно.

Аноним Суб 12 Апр 2014 18:36:28 #72 №109044 

>>109020
Ну это как сказать, вон уже несколько раз сверхновые ебашили.

Аноним Суб 12 Апр 2014 19:20:02 #73 №109048 
1397316002325.jpg

>>109038
Ну это ёбанные звёздные войны какие-то.
А конкретно с Земли какие есть варианты использования оружия земля-космос? Я читал про использование обычных ракет, как тут
http://news.bbc.co.uk/hi/russian/international/newsid_6278000/6278313.stm
но это годится для подавления обычной спутникоты, которая просто летит по заданной траектории. А если спутники противника могут запускать многочисленные противоракеты и маневрировать (а в случае чего ещё и шарахнуть на упреждение)?

Аноним Суб 12 Апр 2014 19:43:19 #74 №109050 

>>109048
>А конкретно с Земли какие есть варианты использования оружия земля-космос?
Добавить к ракете тонну дроби как у "Космического Шотгана"...

А в космосе особо не поманеврируешь. Чтобы увернуться с большой вероятностью нужно ощутимо изменить орбиту. Из-за сильно растянутого во времени маневрового импульса быстро это сделать не получится и противник может без проблем скорректировать курс ракеты.

Аноним Суб 12 Апр 2014 19:45:10 #75 №109051 

>>109043

> Никак не обстоят. Нет.
ну разработки всякого космического термояда все же ведутся

Аноним Вск 13 Апр 2014 03:25:51 #76 №109072 
1397345151751.jpg

>>108967
Два холодных чая этому космогонисту. От Больцмановского мозга и времени возврата Пуанкаре обильно кончал радугой.

Аноним Вск 13 Апр 2014 03:30:28 #77 №109075 

>>109028
Американцы в пятидесятые придумывали космические пушки для сбивания советских военных спутников (и добились, емнип, чуть ли не четверти первой космической). Идея стухла сама собой, ракетой легче будет.

Аноним Вск 13 Апр 2014 11:47:14 #78 №109082 

А куда пропал KSP тред?

Аноним Вск 13 Апр 2014 11:52:47 #79 №109083 

>>109082
Перенес в /vg

Аноним Вск 13 Апр 2014 13:25:01 #80 №109090 

>>109082
Эти дебилы продолжают сидеть в треде после бамплимита и после того как он уплывет с нулевой, пролистай страницы. Ну меньше контента будет, хули, долбоебы же.
мимо оп 20-25 ksp-тредов

Аноним Вск 13 Апр 2014 14:22:42 #81 №109094 

>>108445
Может эта?
http://rghost.ru/54062385

Аноним Вск 13 Апр 2014 14:52:31 #82 №109095 
1397386351454.jpg

Реквестирую книг по аэродинамике и баллистике, в которых как в "Механике космического полета" Левантовского информация разжевана до уровня церковно-приходской школы.

Аноним Вск 13 Апр 2014 17:46:44 #83 №109102 

>>109043
> Скорость неважна, важно ускорение
Ну на радиус поворота влияет. Для веса 1g при 8км/с будет r=8000^2/10=1600км.

Аноним Вск 13 Апр 2014 17:48:44 #84 №109103 

При условии одинаковой высоты над землей.
>>109102

Аноним Вск 13 Апр 2014 18:15:53 #85 №109106 
1397398553871.jpg

Юпитер состоит преимущественно из ядерного топлива. Почему он не воспламеняется от силы трения астрероидов, которые на него падают?

Аноним Вск 13 Апр 2014 18:52:47 #86 №109107 

>>109106
Потому что они падают на него недостаточно сильно.

Аноним Вск 13 Апр 2014 19:01:02 #87 №109109 

>>109107
То есть, это всё равно, что шоркать спичку об сырой коробок?
Но всё же, а если ракетой по нему долбануть с ядерной боеголовкой, то будет какой-либо результат?

Аноним Вск 13 Апр 2014 19:05:08 #88 №109110 
1397401508904.jpg

>>109109
Никакого. И астероиды нихуя не сделают, хоть ты до 0.9999999с разгони. Нужно давление большое. Будь Юпитер в несколько раз больше, у него были бы шансы быть коричневым карликом. Будь он еще во много раз больше, были бы шансы инициировать термоядерную реакцию и стать звездой.

Аноним Вск 13 Апр 2014 22:38:51 #89 №109125 

>>109109
Да, эффект будет. А величина эффекта будет зависеть от мощности заряда.
Начиная от нихуя и заканчивая рапидорашиванием Юпа по всей солнечке.
Однако заставить его гореть в аду самостоятельно - это вряд ли.
Дейтетия с тритием маловато, чтобы превзойти критерий Лоусона.

Аноним Вск 13 Апр 2014 22:48:27 #90 №109127 
1397414907427.jpg

Что за дыры на шаттле?

Аноним Вск 13 Апр 2014 23:14:16 #91 №109131 

>>109127
говно спускать

Аноним Вск 13 Апр 2014 23:25:10 #92 №109132 
1397417110629.jpg

>>109127
Это люки для пушек.

Аноним Вск 13 Апр 2014 23:36:12 #93 №109134 

>>109132
>>109131
Вы заебали петросянить, пидорасы ебливые. Ответьте нормально, сучары мудильные.

Аноним Вск 13 Апр 2014 23:51:31 #94 №109135 
1397418691857.jpg

>>109134
2 секунды в вмеипедии.

Аноним Пнд 14 Апр 2014 00:06:52 #95 №109136 

>>109127
звездный датчик

Аноним Пнд 14 Апр 2014 00:08:39 #96 №109137 
1397419719237.jpg

А вот и про второй люк нашлось.

Аноним Пнд 14 Апр 2014 08:39:44 #97 №109143 
1397450384927.jpg

Аноны, я тут решил предложить вам помечтать вместе со мной.

Насколько возможно, чтобы у планеты вроде нашей с вами Терры получилось наиболее шикарное ночное небо в Галактике?

Ну, например, насколько возможно, чтобы вместо нашей Луны был спутник размерами с Марс, но таки не мёртвый, с серьёзной атмосферой и водой, просто дальше, чтобы приливами не пидорасило громило планету. Спутник спутника было-бы неплохо, да и ещё пару астероидов и что-то вроде Луны. Было бы неплохо, что бы таки в резонансе летали и выпадали довольно часто безлунные ночи. Может ещё Солнечная система была бы поближе к каким звёздным скоплениям, например.

В общем, вопрос у меня из двух частей: возможна ли вышеописанная конфигурация и какой вы видите планету с наикрасивейшим ночным небом в Галактике?
Планета - терра, с гравитацией до 1,2g где-то и природой +- земными

Аноним Пнд 14 Апр 2014 09:38:33 #98 №109144 

>>109143
Да у нас всего лишь из-за Луны до сих пор Елостоны не остыли, а ты говоришь маркс.

Аноним Пнд 14 Апр 2014 14:00:23 #99 №109151 
1397469623460.jpg

>>109143
Это случится, но не сейчас.

Аноним Пнд 14 Апр 2014 14:45:48 #100 №109155 
1397472348681.jpg

Хочу построить самолет. Какие науки мне нужно изучить?

Аноним Пнд 14 Апр 2014 15:00:20 #101 №109156 

>>109155
Очевидно, что физику и математику.
Потом можешь приниматься за ТММ, теормех, материаловедение и сопромат.
Дальше уже будет проще.

Аноним Пнд 14 Апр 2014 16:30:26 #102 №109157 

Оригами для детей и школьников

Аноним Пнд 14 Апр 2014 17:35:32 #103 №109159 

>>109143
Не, слишком наркоманская хуйня и сложная система, при таких размерах спутники спутника не уравновесит себя, ну ты понел.
Я вот из более реалестичного такого думаю о наличии в одной системе 2 обитаемых планет, например. Вместо одной луны - джве, по массе обе примерно равняющихся нашей и движущихся примерно более-менее одинаково вместе.
Хочу наконец увидеть воочию сложную ебанутую систему спутников Плутона.

Аноним Пнд 14 Апр 2014 18:32:14 #104 №109165 

Почему корейцы летали на нашей же Ангаре, а мы нет?

Аноним Пнд 14 Апр 2014 19:59:47 #105 №109167 

Поясните за боцмансковский мозг, что за хуйня такая? Типа разумная планета или как? Вот в статье написано, что мол может случайно сделаться в космосе мозг, абсолютно идентичный человеческому, моему, например, со всему воспоминаниями, но как могут сделаться воспоминания, если я получил их сам входе своей жизни, они не могут сделаться сами. И что за флуктуации, не пойму. Что вы вообще думаете о этой теме.

Аноним Пнд 14 Апр 2014 21:38:48 #106 №109170 
1397497128119.png

>>109167
Лол, да что-то все угорели по хардкору по этой статье, тянке с полчаса объяснял про мозг и время возврата.
Мозг такой может быть устроен как угодно, лишь бы обладал самосознанием.
Воспоминания твои - суть последовательность пространственного расположения нейронов и их связей, ничто не мешает случайному формированию такой последовательности.
Флуктуации - обычные флуктуации виртуальных частиц в вакууме, ничего особенного.

Аноним Втр 15 Апр 2014 01:58:35 #107 №109187 

>>109167
В любой момент времени молекулы воздуха перед твоим носом могут сложиться в слово "Хуй", или высыпанные на стол спички сложатся в уравнение Единого Поля, или из квантовых флуктуаций (виртуальных частиц) спонтанно материализуется тянка (правда, возможно с сиськами на затылке). Ну, ты понел. Нечто может быть закономерным, или случайным. Человеческий мозг - закономерный, возник в результате эволюции (или Б-жественного творения, кому как), а больцмановский - просто так из какой-нибудь хуйни сложился, потому что была такая вероятность. Чистый рандом, короче.
>но как могут сделаться воспоминания
Так это будут ложные воспоминания.

Больцмановский мозг Больцмановский мозг Втр 15 Апр 2014 02:29:18 #108 №109191 
1397514558735.jpg

>>109167
Больцмановсий мозг в этом шее треде три раза - значит правда

Мозг есть система. Причем не простая и не золотая, а где-то очень даже самоподдерживающаяся. Точно так же как мозг не живет долго без тела, тело не живет без экосистемы.

Другими словами, прежде чем возникнет и продолжит существование некая самосознающая система, должна возникнуть система внутри которой возможно появление зайчатков разума.

Это ступенчатый процесс, понимаешь.

Да конечно, возможно, что однажды все быстрые молекулы соберутся в одной половинке жбанки, а медленные - в оставшейся. Вот только в следующую секунду произойдет разброд молекул и возвращение хаоса. Точно так же и с мозгом. Возможно он соткётся из воздуха, и даже осознает себя, но зачем. Ведь в следующюю столетие секунду его не станет.

Поэтому больцмановские мозги предпочитают появляться и осознавать себя а также двачевать капчу на более стабильной, самоподдерживающейся основе.

Вроде, типа того. Так-то.

Больцмановский мозг Больцмановский мозг Втр 15 Апр 2014 02:31:08 #109 №109192 

>>109191
>следующюю
следующую же

быстро-пребыстрофикс

Аноним Втр 15 Апр 2014 11:41:18 #110 №109204 

>>109167
Это мозг, состоящий из всего Космоса. Можно сказать что это мозг Бога.

Аноним Втр 15 Апр 2014 14:59:40 #111 №109213 

>>109204
Исходя из того, что ты родился не мертворожденным, то это очень хуевый мозг.

Аноним Втр 15 Апр 2014 15:06:27 #112 №109215 

Вэйт э сэконд.
Но если ждать очень преочень преочень преочень долго, то через хулиард в хулиардной степени больцмановских мозгов возникнет такой, который просуществует стопицот лет.
Бесконечность - щедрая душа.

Аноним Втр 15 Апр 2014 15:08:46 #113 №109216 

>>109215 -> >>109191

Аноним Втр 15 Апр 2014 15:15:09 #114 №109217 

>>109215
Ну так в этом и суть.

Аноним Втр 15 Апр 2014 15:53:11 #115 №109220 

Вообще, в этом есть что-то жутковатое. Может, у меня под кроватью в это время появился случайно разум, осознал себя, нихуя не понял, ведь у него нет сенсорных систем, и исчез. И никто никогда ничего не узнает.

Аноним Втр 15 Апр 2014 16:59:35 #116 №109227 

>>109156
>физику
>математику
Какие именно дисциплины?

Аноним Втр 15 Апр 2014 22:31:20 #117 №109259 

>>109220
Ну так если об этом никто не узнает, то абсолютно никакой ценности сие событие не представляет.

Аноним Срд 16 Апр 2014 14:11:09 #118 №109329 

>>109259
Ты у мамы антропоцентрист? Не узнает, блджад, никто, блджад. Ценность, у него, блджад. Это твое рождение никакой ценности не представляет, среднестатистическая величина ты, а больцмановский мозг, может, один на всю историю Вселенной, ёба, и никто о нем не узнает. Это, нах, как пропасть без вести, или родиться мамой-алкашкой в мусорном баке, там же окачуриться и сгнить на свалке, даже бомжами не найденным. Ценность, блджад. Или как если бы Иосиф был синяком, третировал Марию и мелкого пиздюка Иисуса, заебошил бы их однажды по белочке топором - и пиздец, и отряд не заметил потери бойца, ведь Б-га нет, он за Иисуса не отомстит, на самом-то деле, пруфов не будет. А ты говоришь - ценность. Ебануться, нах.

Аноним Срд 16 Апр 2014 14:34:21 #119 №109330 

>>109329
Ну не совсем антропоцетрист. Не помню, у Поппера вроде было нечто подобное, типа зачем вообще пытаться строить догадки о происходящем за пределами видимой вселенной, если человеки доказать этого никак не смогут. Или типа никакого значения не имеет тот факт, что в одной из бесконечного числа параллельных вселенных человек с моим именем, моим ДНК-кодом - богатый, успешный, тянки ему дают, и вообще он бессмертный. На моей жизни это никак не отображается. Вот ты пишешь
>один на всю историю Вселенной
А сама история вселенной имеет значение для вселенной? Для пульсара N-3578905 в далекой далекой галактике? Для самого больцмановского мозга, который, не успев себя осознать под кроватью анонима, исчез, не оставив следов? Вообще любая летопись нужна лишь для выявления закономерностей, а понятие закономерности к квантовым флуктуациям не применимо, по крайней мере на данный момент. В этом ведь и суть изучения квантовой механики. Не вздыхать о возможных тысячах появившихся и исчезнувших мозгов во вселенной, а пытаться выявить закономерности теоретически и экспериментально, чтобы следующий мозг уже не прозевать.

Аноним Срд 16 Апр 2014 15:46:02 #120 №109332 

Антропоцентризм заебись. Мы появились во Вселенной которая настроена под нас целиком и полностью, схуяли не быть антропоцентристом? богоцентристом быть чтоле?

Аноним Срд 16 Апр 2014 16:57:21 #121 №109333 

>>109332 Это мы настроились под Вселенную, а не она под нас.

Аноним Срд 16 Апр 2014 17:00:35 #122 №109334 

>>109333 >>109332
Мы сделаны из звездного вещества. Мы - органы чувств космоса. Вселенная нами познаёт себя.

Аноним Срд 16 Апр 2014 19:48:44 #123 №109345 

>>109330
> понятие закономерности к квантовым флуктуациям не применимо
> пытаться выявить закономерности теоретически и экспериментально, чтобы следующий мозг уже не прозевать.
Ты или крестик сними, или трусы надень.
>>109332
> Мы появились во Вселенной которая настроена под нас целиком и полностью
Наука-хуюка говорит, что таки мы под нее настроены. Еще бы сказал, что джунгли настроены под обезьяну, а не обезьяна под джунгли. Эволюция, нах, это тебе не воробьям фигушки показывать, наука работает, суки.
>>109334
> Вселенная нами познаёт себя.
Наркоман несчастный, это как глаза внутрь черепа повернуть.

Аноним Срд 16 Апр 2014 22:06:32 #124 №109353 

Господа, как изменится солнечная система, если Марс разместить в качестве спутника Венеры. Обе планеты получат разогретые пороха и магнитное поле, вулканические газы наполнял атмосферу Марса воздухом, а Венерические дерьмо свяжется и осядет в виде норм почвы. Ведь так?

Аноним Срд 16 Апр 2014 22:32:37 #125 №109356 

>>109353
Дохуя изменится, возможно до фатального для жизни на Земле состояния. Во первых новой системе двойной планеты придется посильнее разогнаться на орбите, чтобы остаться на прежней высоте, и сложно сказать, как это лихая система будет влиять на Землю, пролетая мимо. Во вторых пропажа Марса с прежней орбиты может плохо повлиять на стабильность пояса астероидов.
Мне вот непонятно, почему вы, мамкины терраформеры, так дрочите на перемещение огромных камней в космосе с орбиты на орбиту. Ведь очевидно, что превращение Марса в цветущий сад путем создания на его поверхности всех необходимых механизмов для жизнедеятельности куда менее энергозатратно, нежели перемещение этой каменюки хотя бы на пару километров ближе к Солнцу.

Аноним Срд 16 Апр 2014 23:07:48 #126 №109359 

>>109356
>придется посильнее разогнаться на орбите
С каких это пор масса объекта влияет на орбитальную скорость? Система как крутилась, так и будет крутится.
>как это лихая система будет влиять на Землю, пролетая мимо
Будет в противостоянии слегка притормаживать её вращение и подогревать недра. Возможно что это через пару сотен миллионов лет повысит вулканическую активность Земли на полпроцента.
>пропажа Марса с прежней орбиты может плохо повлиять на стабильность пояса астероидов
Тут хуй знает. Если только разлетятся троянские астероиды Марса а есть ли такие, знает кто? Вообще пояс астероидов довольно стабильная штука, чтобы её взбаламутить нужно что-то потяжелей.
>на его поверхности всех необходимых механизмов для жизнедеятельности
Если на то пошло, то наименее энергозатратно заселять недра недопланет и астероидов.

Аноним Чтв 17 Апр 2014 19:15:33 #127 №109381 

Пришло время окончательно пояснить за пусковую петлю. Почему её не делают и даже не проектируют тестовые образцы? На ссаный быдлокосмический быдлолифт дохуя всяких исследований и конкурсов проводят, хотя он хуже и дороже. Есть какие-то фундаментальные сложности или, как написала загнивающая, у человечества груза не хватит, чтоб это выгодно было пиздец вообще?

Аноним Чтв 17 Апр 2014 20:34:36 #128 №109386 

>>108411
> религиозные американцы
Хилбили из деревень не могут в космос.

Аноним Чтв 17 Апр 2014 20:51:48 #129 №109387 

>>109386
таки могут

Аноним Птн 18 Апр 2014 10:54:17 #130 №109420 

Спейсачеры, как будет выглядеть столкновение центавр или сириусов с земли?

Аноним Птн 18 Апр 2014 11:31:47 #131 №109422 

>>109381
>воздух
>трение

Аноним Птн 18 Апр 2014 14:18:42 #132 №109433 

Анон, если покрасить радиатор из серебра например на гипотетическом КА в фиолетовый не играет роли какой цвет то будет ли он излучать тепло хуже чем неокрашенный? Олсо, какие вещества лучше всего излучают? Т.е лучше всего годятся для радиаторов

Тема [[[email protected]Аноним Птн 18 Апр 2014 15:11:03 #133 №109435 

Космосач нужна помощь видел тут сайт там была расписана жизнь планеты начиная вроде от 2000 года и до 100 000 000 000 вроде так сайт потерял а хотелось бы почитать его

Аноним Птн 18 Апр 2014 20:28:22 #134 №109464 

>>109435
http://futurenow.ru/
Это?

[[[email protected]Аноним Птн 18 Апр 2014 21:51:49 #135 №109472 

>>109464
ООо да спасибо!

Аноним Суб 19 Апр 2014 00:38:54 #136 №109491 

>>109433
>покрасить радиатор
идея - говно. стенке радиатора стоит быть однородной по всей толщине
>лучше всего годятся для радиаторов
любой теплопроводный материал. чем более тпелопроводный - тем лучше

и вообще. радиаторы жидкие внутри и твердые снаружи - полная хрень и прошлый век. только капельные радиаторы. только каплеуловители. только хардкор

Аноним Суб 19 Апр 2014 08:29:59 #137 №109502 

>>109422
Ну откачают, сделают легкие насосы. Что ещё?

Аноним Суб 19 Апр 2014 09:18:46 #138 №109504 

>>109502
Где ты его откачивать собрался? Со всей планеты?

Аноним Суб 19 Апр 2014 15:19:19 #139 №109529 

>>109504
w:
>В основе проекта лежит закольцованный шнур (петля), непрерывно движущийся с огромной скоростью (12—14 км/с) внутри вакуумной трубы.

Аноним Суб 19 Апр 2014 15:27:12 #140 №109532 

>>108409
Вопрос в связи с >>109413.

Из почти что 1,8к эгзопланед, в обитаемой зоне находятся только 2 десятка.

Что за хрень спейсаны? Экзопланетчики филонят?

Аноним Суб 19 Апр 2014 18:39:10 #141 №109560 

>>109529
Ой блять, а лететь где твой груз должен?

Аноним Суб 19 Апр 2014 18:48:21 #142 №109562 

>>109532
Нормальные цифры. Сколько их по твоему должно быть? Миллиарды звезд только в нашем млечном пути и у каждой по планетеа то и несколько. Сколько из них успел исследовать Кеплер?
А ты какие-то жалкие 18к привел.

Аноним Суб 19 Апр 2014 19:24:31 #143 №109566 

>>109560
w:
>а сама петля должна подниматься на высоту до 80 км и держаться на ней за счёт момента инерции вращающегося шнура

Аноним Суб 19 Апр 2014 19:33:20 #144 №109572 

Посоны, есть научпоп передачки про спуск на шатле?

По типу видеорлтд.

Аноним Суб 19 Апр 2014 19:40:12 #145 №109575 

>>108409
Стоит ли смотреть Космос. Пространство и Время™?

Аноним Суб 19 Апр 2014 19:48:09 #146 №109579 

>>109575
Однозначно ДА.

Аноним Суб 19 Апр 2014 22:12:10 #147 №109587 

>>108409
Аноны, скажити пжалста, если кто знает. Есть ли где-то данные по скорости и ускорению разных комических ракет от старта до вывода ПН? Хорошо бы в виде графиков и в сравнении друг и другом.

Аноним Суб 19 Апр 2014 22:40:37 #148 №109589 

>>109572
Ого, их даже выносят на руках после приземления. Они настолько отвыкли от гравитации?

Аноним Вск 20 Апр 2014 01:50:59 #149 №109601 

>>109589
Ты серьезно? Это даже дети знают.

Аноним Вск 20 Апр 2014 13:15:12 #150 №109632 

>>109532
Лучше всего смотрятся массивные планеты на низких и желательно некруговых орбитах. Ясен пень, шансы обнаружить не раскаленный газовый гигант, где год длится пару дней, а планету размером с Землю на приблизительно том же расстоянии от звезды, приблизительно похожей на Солнце - куда меньше.
>>109601
Поляков, Валерий Владимирович, который на своих ногах на спор вышел после 437-дневного полета, не знает. Тренировки, однако.

Аноним Вск 20 Апр 2014 14:41:59 #151 №109639 

Как стать космонавтом?

Аноним Вск 20 Апр 2014 16:10:16 #152 №109646 
1397995816495.jpg

Чем занимаются инженеры-конструкторы, когда все проекты "высушены", а новых конструкторских заданий не поступало?

Платина? Аноним Вск 20 Апр 2014 16:44:41 #153 №109653 
1397997881951.png

>>108409
Посоветуйте книг по астрономии (не астрофизике, а астрономии).
Для изучения с нуля и достаточно доходчивые.

Аноним Вск 20 Апр 2014 16:51:39 #154 №109654 
1397998299124.jpg

>>109653
Что и как наблюдать на небе. Цесевич В.П.

Аноним Вск 20 Апр 2014 17:03:26 #155 №109656 

>>109646
Пьют водку

Аноним Вск 20 Апр 2014 17:05:05 #156 №109657 
1397999105382.jpg

>>109654
Спасибо, как раз то, что я и хотел.
Даже не ожидал такого быстрого ответа.

Аноним Вск 20 Апр 2014 17:40:59 #157 №109667 

А в каком разделе задавать тупые вопросы по авиации?

Аноним Вск 20 Апр 2014 17:54:03 #158 №109671 

>>109667
Можно здесь. Авиация и космонавтика смежны.

Аноним Вск 20 Апр 2014 19:17:33 #159 №109686 

>>109671
Можно ли на самолете без внешних разгонных блоков вылететь на орбиту?

Аноним Вск 20 Апр 2014 19:43:24 #160 №109689 

>>109686
это называется ракетоплан.

пока - нет.

Аноним Вск 20 Апр 2014 20:57:45 #161 №109695 

В каком нынче состоянии находится авиастроение в России? Показывает ли Су-50 и Сухой Суперджет ее возрождение?
Алсо: есть мнение, что китайский J-20 копия МиГ 1.44. Это так?

Аноним Вск 20 Апр 2014 21:03:51 #162 №109696 
1398013431534.webm

>>108409
Ну и где ваша наука теперь? Вакуум-хуякум. Пока вы тут лясы точите, Эйнштейны диванные , анимешные девочки пафосно превозмогают на орбите. Скафандры не нужны, главное - сила воли. Дискас.

Аноним Вск 20 Апр 2014 21:27:17 #163 №109697 

>>109696
> Дискас.
Анус себе подискутируй - двоечник!

Аноним Вск 20 Апр 2014 21:42:41 #164 №109698 
1398015761535.jpg

>>109696
Двачую, ведь никто даже еще не пробовал. А вдруг оно действительно так?

Аноним Вск 20 Апр 2014 21:53:10 #165 №109699 

>>109696
Предлагаю всех даунов-анимуёбов выкинуть в космос без скафандра.

Аноним Вск 20 Апр 2014 21:56:01 #166 №109701 

>>109699
>>109697
Очень конструктивная критика. Вот так вот обращаются в спейсаче со всеми, кто предложит хоть немного смелую идею: назовут школьником и пошлют нахуй. Ни шагу от того,ч то написанно в книжках.

Аноним Вск 20 Апр 2014 21:57:36 #167 №109702 

>>109701
Анимедауна с дошираком вместо мозгов который спизданёт про то, например, что земля плоская, нахуй не пошлют разве что в а и б. Так что уёбывай в один из перечисленных разделов и "траллируй лалок" там.

Аноним Вск 20 Апр 2014 21:59:30 #168 №109704 

>>109702
>разве что в а и б
А мы где по твоему?

Аноним Вск 20 Апр 2014 22:01:13 #169 №109705 

>>109704
ЗАтралил)))

Аноним Вск 20 Апр 2014 22:01:14 #170 №109706 
1398016874156.jpg

>>109696
Я просто в ахуе от всего этого, мне больше нечего сказать

Аноним Вск 20 Апр 2014 22:02:18 #171 №109707 

>>109705
ну-ну, давай-давай

Аноним Вск 20 Апр 2014 22:05:48 #172 №109708 
1398017148924.gif

>>109695
авиасторение в полной перде. надо же понимать что авиастроение было в ссср. и даже в снг. теперь, когда наши, не побоюсь этого слова, партнеры и смежники, оказались в ситуации, в которой продолжение их работы зависит властьпредержащих, считающих их сотрудничество с нами направленным против суверенитета их страны, продолжение такового сотрудничества находится под большим-пребольшим вопросом.
>Показывает ли
Непплохо, непплохо. Это показывает, что Роисся пока еще может в НИОКР. Однако надо понимать, что авиастроение - это не чертежи и не бумажные самолетики. Даже не компьютерные модели. Авиастроение - это авиастроительные заводы, которые либо простаивают, либо выпускают 1 самолет в год.
>китайский J-20
Ну китай всегда славился своим обратным инжинирингом и своими обратными инженеграми. Кроме прочего, моделирование самолетов не является дизайнерской задачей, а является задачей аэродинамики. Законы же аэродинамики одинаковы во крайней мере в россии и в китае.

Аноним Вск 20 Апр 2014 22:28:52 #173 №109709 

Как луна вращается вокруг Земли. Всмыле по какой траектории?

Аноним Вск 20 Апр 2014 22:37:08 #174 №109711 

>>109709
По кругу.

Аноним Вск 20 Апр 2014 22:44:26 #175 №109712 

>>109709
по элебзу
вокруг земли она ОБращается. вращается она вокруг своей собственной оси, как и земляшка, кстати

Аноним Вск 20 Апр 2014 23:00:57 #176 №109714 

Сколько у звезды обычно планет? Какой примерно процент газовых гигантов? Могут ли они быть ближе к звезде, чем твердотельные планеты?

Аноним Вск 20 Апр 2014 23:11:57 #177 №109715 

>>109714
>Сколько
Наличие планетной системы у звезды, скорее правило чем исключение, по нынешним представлениям.
>Какой примерно процент
Да хуй знает. газовые гиганты просто гораздо проще обнаружить.
>Могут ли
запросто, только вот дальние планеты скорее будут ледяыми чем каменными
HD 209458 b например

Аноним Пнд 21 Апр 2014 00:00:40 #178 №109717 

>>109572
На ютубе видел приземление шаттла, вид из кабины. Реально, как на самолете сели. Никаких пафоса, брутальности и превозмогания.

Аноним Пнд 21 Апр 2014 08:24:43 #179 №109729 

>>109572
> видео из орбитера

>>109686
Теоретически, на реактивных двигателях, если они будут достаточно мощны, можно "выпрыгнуть" из атмосферы ненадолго, совершить баллистический полёт вне атмосферы (то есть меньше одного оборота) или даже улететь с планеты (если вторую космическую в атмосфере выдашь). Но чтобы встать на стабильную орбиту, в любом случае нужны движки в космосе, в безвоздушном пространстве, а тягу в таких условиях могут выдавать только ракетные двигатели. Если самолёт будет с ракетными двигателями, то теоретически он сможет встать на орбиту. Но в современных реальных условиях нужно очень много топлива и выгоднее делать ракеты.

Аноним Пнд 21 Апр 2014 08:28:03 #180 №109730 

>>109575
Однозначно нет. Там какую-то хуйню говорят, а про креацианизм ни слова не сказали.

Аноним Пнд 21 Апр 2014 15:07:29 #181 №109744 
1398078449161.png

>>109686
> Можно ли на самолете без внешних разгонных блоков вылететь на орбиту?
На орбиту чего?

На орбиту Земли - нет, ибо придется давать корректирующий импульс в апоцентре, а он будет уже в вакууме. Хотя если очень хочется то можно. Делаем баллистическую траекторию с таким расчетом, чтобы пройти вокруг Луны и совершить гравитационный маневр, ускорившись и подняв перицентр над атмосферой. Способ сферически-вакуумный, естественно, и на практике не прокатит.

На орбиту Солнца - надо быстро разгоняться вертикально вверх. Если успеешь набрать вторую космическую ПЛЮС потери на оставшейся части атмосферы до того как заглохнет движок, то выйдешь на орбиту Солнца. На практике нужна слишком йоба-скорость, тебя распидорасит раньше. Слишком плотная у нас атмосфера.

Аноним Пнд 21 Апр 2014 15:09:28 #182 №109745 
1398078568650.jpg

>>109701
> кто предложит хоть немного смелую идею
Нет, зачем же, таких смелых и прогрессивных людей мы уважаем, просто они живут недолго.

Аноним Пнд 21 Апр 2014 16:43:17 #183 №109756 

>>109696
>превозмогают на орбите.
Но они не на орбите. И превозмогают не силой воли, так и скафандр можно силой воли назвать.

Аноним Пнд 21 Апр 2014 21:00:25 #184 №109764 
1398099625255.gif

>>109717
>как на самолете сели. Никаких пафоса
>шатал Колумбя

>kurwa!111 пиздарики...

Аноним Пнд 21 Апр 2014 22:53:06 #185 №109776 
1398106386963.png

Воспользуюсь своим правом задать тупой вопрос.
Посмотрите на пикрилейтед. Почему в его взгляде сталь, а в оса
Первая группа каракулей символично изображает использующуюся с давних времен вытеснительную систему подачи топлива. Для тех, кто не знает: это когда вместе с баками окислителя и горючего цепляют ещё и канистру с гелием, который и выдавливает горючку наружу давлением.
Вторая схема безупречно показывает все аспекты работы подачи топлива с помощью ТНА. Насос, работающий за счет той же горючки, или просто от электричества, высасывает соляру, что твой дед из колхозного КамАЗа в голодную пору.

Вопрос задам сразу: а как еще можно подавать топливо в камеру сгорания?

Сегодня вечером я задался этим вопросом и подумал - а почему бы не выдавливать? Правда, деформирующиеся аки тюбик баки - затея весьма незаурядная. А почему бы не использовать поршни, подумал я, и наваял третью схему.

Предлагаю подискутировать на эту тему.
Сразу же скажу очевидные минусы, и попробую контраргументировать:
1. утечки топлива между стенками и поршнем: уплотнительные кольца
2. как ты уместишь поршень чтобы он раскрылся на всю длину? ну тот же ножничный механизм, например.
3. масса - компенсируется ненадобностью брать с собой ТНА, вытеснительные баки, моторы осаждения и прочую хуйню-малафью. Дай только электринчество. Особенно удобно при использовании самовоспламеняющейся пары типа НДМГ+АТ.
В общем идеальный вариант для РБ дальнего плавания.

Дискасс го.

Аноним Пнд 21 Апр 2014 23:59:51 #186 №109784 

>>109776
> 1. утечки топлива между стенками и поршнем: уплотнительные кольца
Удачи провернуть это с водородом, например.

Аноним Втр 22 Апр 2014 00:01:13 #187 №109785 
1398110473683.gif

>>109776
1. Протекать будет, как ни уплотняй.
2. Внутренняя поверхность бака отнюдь не ровная, а с ребрами для предотвращения шатания топлива и окислителя с формированием воронки.

Кроме того, даже при решении этих проблем, все равно непонятно, чем это лучше банальной вытеснительной схемы - наличие движущихся частей, как минимум не более высокое давление, вряд ли легче будет.

Аноним Втр 22 Апр 2014 00:07:44 #188 №109786 

>>109784
НДМГ, например. Или керосин.

>>109785
Я знаю про внутреннюю поверхность. Только в случае поршня там не нужны будут ребра для избавления от воронок, ведь баки всегда будут полны (их объем будет уменьшаться с поршнем).

Вытеснительная система - это какой-то сжатый газ. А тут непосредственно выдавливание поршнем.
Все равно, что пытаться закачать воздух в колесо из воздушного шарика или накачать его насосом. Хуевый пример, но мне похуй.

Аноним Втр 22 Апр 2014 02:37:31 #189 №109793 

>>109786
>А тут непосредственно выдавливание поршнем.
Утяжеление и усложнение конструкции. Нахуй не надо такое.

Аноним Втр 22 Апр 2014 03:48:21 #190 №109794 

Почему не делают сверхсветовой движитель? Нлошки вон с тау кита прилетают же.
Почему на марсоходы не ставят модули сверхветовой связи? Управлять с пингом в десятки минут не очень удобно же.
Почему не делают синтетическое мясо? Питаться жертвами и трупами дикарство же, космические цивилизации смотрят от этого на нас как на говно.

Аноним Втр 22 Апр 2014 04:56:16 #191 №109798 

>>109794
Ты откуда к нам такой? Из /sf/, или из дома скорби?

Аноним Втр 22 Апр 2014 11:05:44 #192 №109802 

>>109793
> усложнение конструкции.
Будто турбонасосный агрегат это дохуя простая конструкция.
Кстати, поршневая система запросто подразумевает дозаправку баков.

Аноним Втр 22 Апр 2014 11:11:03 #193 №109803 

>>109794
>Почему не делают сверхсветовой движитель?
Делают: наса думает, что можно получить сверхсветовое движение если упасть в пузырь альбукерке.
>Нлошки вон с тау кита прилетают же.
Голословное утверждение.

>Почему на марсоходы не ставят модули сверхветовой связи?
Потому, что мы не умеем в сверхсветовую связь даже теоретически.
>Управлять с пингом в десятки минут не очень удобно же.
А что поделаешь во вселенной, в которой взаимодействие распространяется со скоростью света?

>Почему не делают синтетическое мясо?
Делают, ведутся изыскания.
>Питаться жертвами и трупами дикарство же, космические цивилизации смотрят от этого на нас как на говно.
Еще одно голословное утверждение.

Аноним Втр 22 Апр 2014 11:55:09 #194 №109808 

>>109802
>Будто турбонасосный агрегат это дохуя простая конструкция.
У ТНА основная функция не топливо сосать из баков, так, на всякий случай напомню. Ты же предлагаешь прихуярить ЙОБУ сверху только для того, что бы давить на поршень. Кстати, шток поршня у тебя будет над ракетой торчать? Или ты еще чего придумал? И расскажи, какой агрегат будет этот поршень двигать.

>запросто подразумевает дозаправку баков.
Прямо в полете!

Аноним Втр 22 Апр 2014 12:17:50 #195 №109810 

>>109808
> И расскажи, какой агрегат будет этот поршень двигать.
Собственным весом. Во время разгона его вес увеличится и будет создавать давление больше, чем само топливо, ибо плотность поршня больше, чем у топлива.

Аноним Втр 22 Апр 2014 13:40:44 #196 №109812 

>>109810
>Собственным весом
Ты предлагаешь положить в ракету нахуй не нужный груз, я все правильно понял?

Аноним Втр 22 Апр 2014 13:56:11 #197 №109817 
1398160571140.jpg

Кстати, Аноны, раз уж тут опять пузырь АльбукеркеАлькубьерре всплыл, можете рассказать как там в НАСА с эксперементами и подтверждениями? Они, вроде, собирались на тысячные(или миллионные) доли процента пространство искривлять, нет?

Аноним Втр 22 Апр 2014 13:56:35 #198 №109818 

>>109803
>Голословное утверждение.
В какой крайности догматического мышления ты, в атеистической или религиозной?

Аноним Втр 22 Апр 2014 14:01:04 #199 №109819 

Кстати, Аноны, раз уж тут опять кривое пространство всплыло, можете рассказать почему пространство отождествляют с полем? Они, вроде, разные сущности, зачем тогда говорить искривляю пространство если искривляешь поле, нет?

Аноним Втр 22 Апр 2014 14:14:34 #200 №109820 

>>109812
Почему не нужный? Он будет создавать давление в баке.

Аноним Втр 22 Апр 2014 14:31:44 #201 №109822 

>>109817
Да это попил-откат очередной, сиди на своей земляшке и не рыпайся.

Аноним Втр 22 Апр 2014 15:43:46 #202 №109826 
1398167026876.jpg

>>109818
>религиозной
Религиоблядям место в /re/. Атеизм тут ни при чем. Есть доказательства прилетов инопланетян с Тау Кита? Нет - уебывай.
Есть у тебя способ путешествовать быстрее с? Хотя бы уравнениях? Нет - уебывай.
Короче, забирай свою наркоманию и уебывай, на все твои тупые вопросы тут ответили. И вести дискуссии с больными на голову тут никто не будет. Для этого есть более илитарные доски.

Аноним Втр 22 Апр 2014 15:45:48 #203 №109827 

>>109820
Но у нас есть более эффективные и технологичные способы создания дОвления в баках, отличающиеся от от привычного тебе шприца, наркоман.

Аноним Втр 22 Апр 2014 16:35:53 #204 №109833 
1398170153872.jpg

>>109827
Можно шарик с топливом выжимать моторчиком. Заодно топливо сделать желеобразным, чтобы при авариях меньше растекалось.

Аноним Втр 22 Апр 2014 16:37:45 #205 №109834 

>>109819
Не пространство, а пространство-время. А поле это материя.

Аноним Втр 22 Апр 2014 16:41:01 #206 №109835 

>>109822
Так это гипердрайв же, на планетах не работает. Так что придется валить отседова на пердячем ходу все равно. И даже из системы, скорее всего.

Аноним Втр 22 Апр 2014 17:31:13 #207 №109837 
1398173473984.jpg

>>109833
Ты можешь объяснить, зачем? Какие сложности решают твои нововведения?

Твой поршень облегчает конструкцию?
Нет, утяжеляет ее.

Желеобразное топливо будет таким же эффективным, как и обычное?
Нет, не будет.

Аноним Втр 22 Апр 2014 19:26:50 #208 №109842 

>>109837
Поршень не мой, а желеобразное топливо вполне себе более эффективно, в него можно алюминия или лития напихать например. А задача пошевелить мозговой извилиной, think different. Чиста чтобы не сидеть в пещерах на паровозах со своей СТАБИЛЬНОСТЬЮ, в то время, как частники захватывают космос. Хотя для тебя это всего лишь игры разума, не более.

Аноним Втр 22 Апр 2014 19:46:00 #209 №109846 

>>109842
>в то время, как частники захватывают космос.
Частники летают на поршневых ракетах с желеобразным топливом. Шевеля извилинами не получи перелом мозга.

Аноним Втр 22 Апр 2014 19:51:31 #210 №109847 

>>109846
Ну хочешь, будет газообразное топливо и разгон на гиперзвуковых дирижаблях. Ты какой-то унылый, жизнь это не только бесконечное клонирование пятидесятилетней ракеты!

Аноним Втр 22 Апр 2014 20:01:29 #211 №109850 

>>109847
>Ты какой-то унылый, жизнь это не только бесконечное клонирование пятидесятилетней ракеты!

Я только за, пусть будут гиперзвуковые дирижабли, ты главное дай материалы, при которых дирижабли не развалятся, и двигатели для дирижаблей, которые будут их разгонять до сверхзвука. А главное, что бы это было эффективнее, чем пятидесятилетняя ракета.

>Ты какой-то унылый
Тебя отлично повеселят в /sf/, вот там-то настоящая жизнь, а в /sci/ тебя еще и эфиром накормят - забалдеешь.
А так, да, тут унылые люди поклоняются Циолковскому и Королеву, да поддрачивают на изделия Фон Брауна, но только осторожно, под одеялом, пока никто не видит.

Аноним Втр 22 Апр 2014 23:59:58 #212 №109870 
1398196798755.jpg

>>109847
>разгон на гиперзвуковых дирижаблях
Я бы не против и обычных. Какого хрена я никогда не смогу пересечь Атлантику на борту летающей йобы объёмом в сто тысяч кубов. У меня ГИНДЕНБУРГ от такой хуйни.

Аноним Срд 23 Апр 2014 00:25:22 #213 №109871 

>>109870
>Какого хрена я никогда не смогу пересечь Атлантику на борту летающей йобы объёмом в сто тысяч кубов.

Что-то мне подсказывает, что ты и на современном самолете Атлантику не пересечешь. Ибо на билет нужно деньги, а ты на двачах сидишь.

Аноним Срд 23 Апр 2014 01:41:31 #214 №109880 
1398202891994.jpg

>>109871
Хреновая из тебя Ванга. Скайсканер сообщает, что билет из Москвы в Лос-Анджелес, например, обойдётся мне в 21 тысячу рэ. Лично мне хватит. А вот в йобанную Гавану билеты почему-то вдвое дороже.

Аноним Срд 23 Апр 2014 05:47:00 #215 №109895 

>>109880
Я хотел постебаться, а вышло, что ты такой же, как я. Мечтающий о Кубе.

Аноним Срд 23 Апр 2014 09:19:59 #216 №109896 
1398230399815.jpg

>>109895

Аноним Срд 23 Апр 2014 13:22:32 #217 №109907 

>>109696
Человек без кислорода жить не может,(да, проверяли)

Аноним Чтв 24 Апр 2014 10:42:25 #218 №109961 

1)Почему грят, что на марсе была вода в жидком виде, если солнце тогда светило намного тусклее, и если даже сейчас на марсе совсем небольшой плюс по цельсию бывает, то тогда и подавно не могло быть? Неужели парниковый эффект может быть настолько сильным?
2)Насколько далеко от современного солнца может быть плюсовая температура (в зените) на планетах (у которых нет внутренних источников тепла)? Вот например, если взять планету из какой-нибудь сажи с фреонной атмосферой.

Аноним Чтв 24 Апр 2014 10:59:16 #219 №109963 

>>109961
Я слышал что на Венере парниковый эффект прибавляет 500 градусов температуры.

Аноним Чтв 24 Апр 2014 12:10:39 #220 №109964 

>>109961
>2)Насколько далеко от современного солнца может быть плюсовая температура (в зените) на планетах
Очень далеко, дальше, чем Плутон намного дальше.

Аноним Чтв 24 Апр 2014 12:31:17 #221 №109966 

>>109964
а как считать? просто взять Закон Стефана — Больцмана, подставить туда полную мощность солнца, 274К и через площадь найти радиус?

Аноним Чтв 24 Апр 2014 12:40:43 #222 №109968 

>>109964
Ну по этой формуле (точнее, через эффективную температуру) получилось 316млн км вроде.
>>109966

Аноним Чтв 24 Апр 2014 16:51:07 #223 №109978 

>>109961
> Неужели парниковый эффект может быть настолько сильным?
Венерианский суперкритический кисель вместо атмосферы передает привет со своими 90 бар и 470°C.

Аноним Чтв 24 Апр 2014 19:43:47 #224 №110002 

Антон, а можно ли спускать всё лишние тепло от двигателей и прочих систем в ЧД? Она же вроде как не выпускает свет(т.е фотоны), а ИК излучение - фотоны. И можно ли построить КА из вещества НЗ?

Аноним Чтв 24 Апр 2014 22:09:52 #225 №110012 

>>110002
>можно ли спускать
можно, но только до первого поворота. Когда ты полетишь в налево, например, а твой "холодильник" продолжит равномерное прямолинейное движение.
>КА из вещества НЗ?
Если не платина, то золото. Нет, низзя, бабахнет. Вместе с твоим пердаком, видимо.

Это если не считать того, что на земляшке "нейтронное вещество" еще долго получено не будет, а выковырять его из НЗ не представляется возможным.

Аноним Птн 25 Апр 2014 00:22:55 #226 №110018 

ВОПРОС
ЗАЧЕМ НАМ ЛУННАЯ БАЗА?
чтоб нас не кокнули?

Аноним Птн 25 Апр 2014 00:43:14 #227 №110021 

>>110018
за тем же, за чем и МКС, только основательнее
плацдарм для зогвоевания маркса

Аноним Птн 25 Апр 2014 01:01:55 #228 №110028 

>>110018
Много науки и космодром подскока к Марсу.

Аноним Птн 25 Апр 2014 06:20:26 #229 №110054 

Какое нужно увеличение чтобы увидеть щель кассини? А марс чтобы рассмотреть?

Аноним Птн 25 Апр 2014 10:59:46 #230 №110058 

>>110054
атмосфера нужна ровная

Аноним Птн 25 Апр 2014 11:23:34 #231 №110063 

Кто сказал свету двигаться с определенной скоростью? почему не меньше и не больше? (с учетом погрешностей измерения конечно же), значит ли это что есть какая-то сила которая раздает все эти законы?

Аноним Птн 25 Апр 2014 12:01:42 #232 №110066 

Анон, ищу фильмы про космос.Сквозь горизонт годный,
про Марс 2 фильма высадку - сойдет, про камни-убийцы на Луне не надо

Аноним Птн 25 Апр 2014 13:09:59 #233 №110071 

>>110063
> значит ли это что есть какая-то сила которая раздает все эти законы?
Электрическая и магнитная постоянные.

Аноним Птн 25 Апр 2014 13:17:31 #234 №110073 

>>110066
Пекло.
Если повеселее - светлячок (сериал).

Аноним Птн 25 Апр 2014 14:04:06 #235 №110078 

>>110012
>огда ты полетишь в налево, например, а твой "холодильник" продолжит равномерное прямолинейное движение
Магнитное поле + чд которая вращается и имеет магн. поле

Аноним Птн 25 Апр 2014 14:05:34 #236 №110079 

>>110012
>бабахнет
Можешь пояснить? От чего БАБАХНЕТ?

Аноним Птн 25 Апр 2014 14:32:56 #237 №110081 
1398421976169.jpg

Я всё правильно делаю?

Аноним Птн 25 Апр 2014 14:40:31 #238 №110082 

Анон что я увижу если приземлюсь на НЗ с пикчами желательно? Почему есть куча материалов по ЧД, но по НЗ нихуя нет?

Аноним Птн 25 Апр 2014 15:39:13 #239 №110088 
1398425953397.png

>>110082
>Почему есть куча материалов по ЧД, но по НЗ нихуя нет?
Про НЗ материала гораздо больше, чем про ЧД, ибо изучены они гораздо лучше.

>что я увижу
Ничего, ты умрешь гораздо раньше.

>если приземлюсь на НЗ

Тебя распидорасить мощнейшим гравитационным полем еще на подлете.
Если это будет пульсар, то тебя зажарит радиацией гораздо раньше, чем распидорасит гравитационным полем.
Если это будет магнитар, то он еще за несколько тысяч километров высосет из твоей крови все железо и ты снова умрешь.
Так вот, когда то, что осталось от твоей тушки ебнется на поверхность, ты будешь представлять из себя блин в пару атомов толщиной.
Но скорее всего ты испаришься в горячей углеродной атмосфере нейтронной звезды в 10 сантиметров толщиной.
При любом раскладе тебе пиздец.
Приятной прогулки.
И не приведи тебя Эйнштейн попасть в тесную двойную систему НЗ-НЗ, или НЗ-звезда, там такой праздник Сотоны происходит, что лучше держаться от этой хуйни подальше

Аноним Птн 25 Апр 2014 16:11:22 #240 №110089 

>>110088 Блжад, я знаю что меня распидорасит радиацией и гравитацией. Допустит, что я Аллах и абсолютно бессмертен и могу комфортно ходить по поверхности НЗ. Что я увижу? Как искривится пространство гроветация же. Как выглядит рельеф? Что я увижу на "небе"?

Аноним Птн 25 Апр 2014 16:18:58 #241 №110090 

http://epizodsspace.no-ip.org/bibl/u-t/1969/lazer.html Взлетит? Вообще фотонный двигатель из лазера/кучи светодиодов/другого источника света реален? Что лучше всего подходит в качестве источника фотонов? Всякие йоба гамма и рентген не рассматриваем. Только видимый спектр, ИК и УФ, только хоркор

Аноним Птн 25 Апр 2014 16:50:21 #242 №110091 

>>110089
>Как выглядит рельеф?
Никак. Самый высокий "горный хребет" будет в полмиллиметра высотой.

Аноним Птн 25 Апр 2014 18:04:08 #243 №110093 

>>110089
>могу комфортно ходить по поверхности НЗ
>Что я увижу на "небе"?
Яркий гамма-свет. В атмосфере НЗ происходят реакции термоядерного синтеза, как в ядре обычной звезды. Возможно не повсеместно, а местами и не все время, а как накопится свежего водорода или геля.

Аноним Птн 25 Апр 2014 18:14:47 #244 №110094 

>>110090
>фотонный двигатель из лазера/кучи светодиодов/другого источника света реален?
Да, твой фонарик уже фотонный двигатель. Но не взлетит, потому что mv=MV, но одновременно e=mv^2. Литературно перефразируя, при росте удельного импульса потребная энергия растет квадратично. Реактивные двигатели с высоким удельным импульсом удручающе неэффективны в энергетике, и это принципиально.

Аноним Птн 25 Апр 2014 18:15:52 #245 №110095 

>>110094
>e=mv^2.
пополам, конешно.

Аноним Птн 25 Апр 2014 18:39:26 #246 №110100 

>>110066
Путешествие на европу, гравитация, затерянные в космосе

Аноним Птн 25 Апр 2014 18:45:05 #247 №110101 

>>110100
Europa report, a не путешествие на европу.

Аноним Птн 25 Апр 2014 20:06:46 #248 №110106 

>>110091 Это из за гравитации?
>>110093 Но ведь я не могу видеть в гамме. И атмосфера нз толщиной в пару сантиметров, а я в 201 см. Значит я буду выше атмосферы.
Я буду видеть искривлённое пространство?

Аноним Птн 25 Апр 2014 20:22:48 #249 №110108 

>>110094 А что если я буду светить йоба лазером на борту КА на солнечный парус прикрепленный к КА? И профит, и не надо тратить кучу энергии

Аноним Птн 25 Апр 2014 22:55:14 #250 №110126 

Антуаны, я тут ПОКУМЕКАЛ. НЗ же вращаются со скоростью в дохуя оборотов так ведь? (вплоть до пары тысяч в секунду). А что если центробежная компенсирует гравитацию на экваторе я думаю с парой тысяч оборотов в секунду это вполне реально? Тогда можно же спокойно на неё приземлится хотя я не знаю что там с радиацией не на пульсарах, но если на поверхности обычных НЗ тоже ГРОБ ГРОБ КЛАДБИЩЕ с радиацей и магнитными полями то тупо посадит робота и исследовать, так ведь? Понятно, что посадку мягкой сделать не удастся но приземлится ведь можно будет?

Аноним Суб 26 Апр 2014 03:57:26 #251 №110146 

>>110126
> ГРОБ ГРОБ КЛАДБИЩЕ с радиацей и магнитными полями то тупо посадит робота
То-то даже японцы не смогли роботов в Фокусиме приспособить.

Аноним Суб 26 Апр 2014 05:54:03 #252 №110147 

>>110108
Та же хрень, твой йоба-лазер будет йоба-тяжелый из-за топлива, его разгон потребует больше йоба-энергии, которая потребует больше топлива, и так заново по формуле циолковского. А вот если отцепить лазер, оставить его на орбите Солнца, то можно им светить на легкий парус, такие есть проекты.

Аноним Суб 26 Апр 2014 05:57:37 #253 №110148 
1398477457288.jpg

>>110146
Робота можно не электронного, а пневматического например.

Аноним Суб 26 Апр 2014 10:31:32 #254 №110150 

Или же собрать аналоговый йоба компуктер. Ссылкарилейтед например. Оптический процессор. Аналоговые компоненты не боятся магнитных полей и радиации в отличии от кремниевых собратьев
http://www.extremetech.com/computing/121587-ibm-creates-cheap-standard-cmos-1tbps-holey-optochip

Аноним Суб 26 Апр 2014 12:01:03 #255 №110153 

Допустим, через очень много-много лет человечество создаёт аппарат, двигающийся со скоростью хотя бы 50 тысяч километров в секунду. До скорости света фантазировать не будем. Как на таких скоростях путешествовать в космосе, если там дохуя всякой херни летает? Херня эта, конечно, в основном мелкая, но её будет же реально много, а при таком быстром движении удар будет вообще страшным.
Чем защищаться от повреждений при столкновениях? Прикреплять сверхпрочный щит размером с солнцезащитный из фильма «Пекло» + обеспечивать систему самовосстановления?

Аноним Суб 26 Апр 2014 12:21:36 #256 №110155 

>>110153
>Как на таких скоростях путешествовать в космосе, если там дохуя всякой херни летает?
Мелкую хуйню электризовать встречным потоком плазмы и собирать магнитным полем для пополнения бортовых запасов рабочего тела, а от крупной хуйни самое эффективное средство - молитва.

sageАноним Суб 26 Апр 2014 13:28:17 #257 №110158 

>>110150
> Оптический процессор
> is basically a conventional silicon chip
> vertical-cavity surface-emitting lasers
> Photodiodes
> CMOS
Ты ведь не читал свою ссылку, да?

Аноним Суб 26 Апр 2014 13:30:07 #258 №110159 

>>110155
Закон сохранения импульса вежливо претендует на твою попку:
http://ru.wikipedia.org/wiki/Межзвёздный_прямоточный_двигатель_Бассарда#.D0.9E.D0.B3.D1.80.D0.B0.D0.BD.D0.B8.D1.87.D0.B5.D0.BD.D0.B8.D0.B5_.D0.BD.D0.B0_.D1.81.D0.BA.D0.BE.D1.80.D0.BE.D1.81.D1.82.D1.8C

Аноним Суб 26 Апр 2014 13:37:23 #259 №110160 

>>110153
Помимо маршевого двигателя у нас будет более слабый фронтальный, выжигающий среду впереди корабля струей плазмы (если скорость корабля невелика) или гамма-излучением (если движок фотонный и нужны большие скорости). И не понадобится толстый щит.

Отличие от >>110155 в том, что встречное вещество нужно разгонять с пути корабля, придавая ему поперечную скорость, а не просто ионизировать и собирать. Таким образом, дезинтегратор будет немного уменьшать итоговое ускорение, но на фиксированную величину, а не на возрастающую, как при трении о среду (или, что эквивалентно, как при улавливании ионов).

Аноним Суб 26 Апр 2014 15:18:23 #260 №110166 

>>110066
Аполлон 13 наверни, еще захочешь.

Аноним Суб 26 Апр 2014 15:18:53 #261 №110167 

А что, если Земля будет крутиться так, что все объекты на её поверхности будут двигаться с первой космической скоростью?

Аноним Суб 26 Апр 2014 16:47:25 #262 №110169 
1398516445510.jpg

>>110167
Из-за компенсации гравитации высвободится огромное количество механической энергии, запасенной в горных породах (эффект освобожденной пружины). Опять же, на подкорковые слои перестанут давить кора и магма, и огромные массы глубинных горных пород превратятся в магму же (ранее для этого было слишком большое давление). Поверхность Земли будет уничтожена, облако пыли, обломков и вулканических газов скроет обнажившуюся жидкую мантию. Планета-ад, короче.

Аноним Суб 26 Апр 2014 17:40:57 #263 №110173 
1398519657054.jpg

>>110169
>>110167
На самом деле не обязательно даже до первокосмической розгонять, достаточно половины, например. Как следствие - образование новых тектонических разломов из-за изменения формы геоида. Вода переместися тож, деревню Гадюканданг в Индонезий смоет нахуй. Обледенение в арктике и антарктике, новые острова там. Это если не считать что атмосфера поредеет.

Если же продолжать банкет и раскрутить до первой комической, на экваторе подпрыгнешь - и все, с концами. хохлов - туда

Но тащемта, нельза просто так взять и раскрутить земляшку. Из-за упомянутого изменения геоида, будет увеличаваться момент инерции геоида, в что потребует моара и моара энергии.

И вопщи, земляша - это не каменный шар, который можно взять за поверхность ручечкой и раскрутить. Это что-то навроде не вареного курьего яйца - раскручиваешь поверхность, а мантия с астеносферой тормозят все веселье.
Еллустоны просыпаются опять жи.

>что если наша планета
Не думаю. Чистилище - да.

Аноним Суб 26 Апр 2014 17:44:56 #264 №110174 

>>110173
Осло при первой космической на эквавторе - сатурноподобные кольца вокруг Земляшки из пемзы и туфа.

Аноним Суб 26 Апр 2014 17:49:24 #265 №110175 

>>110153
Кораболь из астероида

Аноним Суб 26 Апр 2014 17:58:19 #266 №110177 

>>110173
> Это что-то навроде не вареного курьего яйца - раскручиваешь поверхность, а мантия с астеносферой тормозят все веселье.
А при торможении выделяется энергия, которая хорошо проплавляет и недра и кору. Результат тот же - планета-ад.

Аноним Суб 26 Апр 2014 17:59:15 #267 №110178 

Бетельгейзе.
Когда она станет сверхновой?

Отчего происходят взрывы нейтронных звёзд? откуда берётся такое охуенное количество энергии?

Аноним Суб 26 Апр 2014 17:59:30 #268 №110179 

>>110173
> Не думаю. Чистилище - да.
Буддист штоле?

Аноним Суб 26 Апр 2014 18:00:47 #269 №110180 

>>110178
> Отчего происходят взрывы нейтронных звёзд?
Ну, наверно, это поверхностные термоядерные взрывы в скопившейся вокруг НЗ плазме.
А еще, поговаривают, возможны столкновения нейтронных звезд, которые дают характерные гамма-всплески.

Аноним Суб 26 Апр 2014 18:08:57 #270 №110182 

>>110180
Насколько знаю, термоядерная реакция с выделением энергии идёт при создании элементов легче железа. Всё что тяжелее-синтезируется с поглощением энергии.
Может звезда досинтезироваться до урана 235 и потом уебать?

Аноним Суб 26 Апр 2014 18:50:14 #271 №110183 

>>110179
>Буддист
Да ну. Просто надеюсь на призовую игру. еще бы память не стирали каждый раз

Аноним Суб 26 Апр 2014 19:07:12 #272 №110185 

>>110182
Есть мнение, что синтез тяжелых элементов годный "холодильник" во время взрыва сверхуновой.
Но получившиеся тяжелые элементы рассеиваются, а концентрируются только гравитационно в ядрах планет.

Так что нет, ибо самопроизвольный атомный взрыв возможен только на самопроизвольно создавшемся заводе по обогащению делящихся материалов, что представляется КРАЙНЕ маловероятным.

Аноним Суб 26 Апр 2014 19:09:36 #273 №110186 

>>110185
Осло, естественный атомный реактор возможен не только в ядре земляшки, но даже вблизи поверхности.
http://ru.wikipedia.org/wiki/%D0%9F%D1%80%D0%B8%D1%80%D0%BE%D0%B4%D0%BD%D1%8B%D0%B9_%D1%8F%D0%B4%D0%B5%D1%80%D0%BD%D1%8B%D0%B9_%D1%80%D0%B5%D0%B0%D0%BA%D1%82%D0%BE%D1%80_%D0%B2_%D0%9E%D0%BA%D0%BB%D0%BE

Аноним Суб 26 Апр 2014 19:09:38 #274 №110187 

>>110158 Читал. Я говорю о полностью аналоговом. Это привёл прост))

Аноним Суб 26 Апр 2014 20:13:12 #275 №110192 

>>110183
>еще бы память не стирали каждый раз
Записывай же. комикс фрифолл читай

Аноним Суб 26 Апр 2014 20:21:04 #276 №110193 

>>110186
>естественный атомный реактор возможен
Был возможен 2млрд лет назад. Сейчас от того урана-235 в наблюдаемой вселенной осталось пополам пополам и пополам, шестая часть примерно. На планете Земля естественно тоже.

Аноним Суб 26 Апр 2014 20:28:43 #277 №110194 

>>110193
Возможен и сейчас. Есть и более крупные/концентрированные месторождения, но возникнуть естественным образом реакция может только в силу случая, как в Окло.

Аноним Суб 26 Апр 2014 20:32:21 #278 №110196 

>>110187
Но почему ты решил, что по твоей ссылке не аналоговый компьютер? Лазеры, фотодиоды и КМОП вполне могут быть его составляющими.

Аноним Суб 26 Апр 2014 20:36:30 #279 №110197 

>>110194
Реактор Окло, будучи запущен сейчас, уничтожит выбросами наземных млекопитающих на планете. Мне не нравится такая возможность.

Аноним Суб 26 Апр 2014 21:44:12 #280 №110199 

сап. Дочитал сейчас Хоккинга, - что можно почитать еще в таком духе или на 1 уровень повыше?

Аноним Суб 26 Апр 2014 22:01:43 #281 №110202 
1398535303204.jpg

>>110199
Пенроуз, Грин.

> на 1 уровень повыше
Ландафшиц. Ибо это только на словах популяризаторов все так просто

Аноним Суб 26 Апр 2014 22:03:22 #282 №110203 

>>108409
> И снова с вами — лучший, после огурцов, тред спейсача!
Я считаю, что это худший и самый вредный тред спейсача, ибо большинство задает здесь вопросы, достойные отдельного треда.

Аноним Суб 26 Апр 2014 22:10:27 #283 №110205 

Блять, столько лет считал, что у чд гравитация на горизонте численно равна скорости света. Так это или нет?
И почему для поверхности НЗ какой-то пиздец тут выходит? Выходит около 10^11 м/с2.

Аноним Суб 26 Апр 2014 22:11:20 #284 №110206 

>>110185
>Есть мнение, что синтез тяжелых элементов годный "холодильник" во время взрыва сверхуновой.

Так оно и получается, синтез тяжёлых (железо и тяжелее) элементов идёт с поглощением энергии.

Мне шаблон рвёт то, что сверхновая получается из выгоревшей звезды, у которой по сути и энергия-то вся выработана. "догорела" звезда до железа - дальше гореть-то нечему, с чего такой шикарный фейерверк как взрыв сверхновой ?
И шарахает неслабо, в 40 световых годах атмосферу с планет сдувает.

Аноним Суб 26 Апр 2014 22:17:12 #285 №110209 

>>110126
Ну хоть в формулы-то простейшие покумекай немного. Я посчитал, что максимальная разница будет в 1/5 скорости света, между тем как на поверхности гравитация около 2/3.
Вообще Сергей Попов у нас НЗ занимается, много лекций рассказывает, гуглабельно.

Аноним Суб 26 Апр 2014 22:18:35 #286 №110210 

>>110206
Так там железо - продукт реакции. А сама реакция с кислород-углеродной смесью.

Аноним Суб 26 Апр 2014 22:22:17 #287 №110212 

>>110205
Вопрос тупой, в буквальном смысле, ибо задан некорректно: ты сравниваешь гравитацию в численном выражении (чзх?) со скоростью.

Аноним Суб 26 Апр 2014 22:23:47 #288 №110213 

>>110212
Наверно, он про скорость убегания

Аноним Суб 26 Апр 2014 22:29:46 #289 №110217 

>>110209
Да даже без релятивистстких поправок вещество звезды на экваторе тоже будет вращаться с первой космической и не будет удерживаться в самой звезде.

Аноним Суб 26 Апр 2014 23:59:54 #290 №110228 

Космоанон, посоветуй какую книгу, посвященную биографии Королева. Хочу обмазаться

Аноним Вск 27 Апр 2014 00:08:35 #291 №110229 

>>110217
а какова первая космическая у нз?

Аноним Вск 27 Апр 2014 00:10:06 #292 №110230 

>>110212
Ускорение свободного падения и скорость света.

Аноним Вск 27 Апр 2014 00:38:21 #293 №110231 

>>110230
А зачем ты численно сравниваешь ускорение и скорость? Нужно сравнивать скорость со скоростью, и ускорение с ускорением. Совсем, блджад, школота в физические величины не может, раньше не было такой хуйни, Сталина на вас нет!!!1

Аноним Вск 27 Апр 2014 00:52:36 #294 №110232 

>>110231
Да ёпт, обязательно занудство это? Нечего ответить - отвечай вопросом "зачем, нафига, чё ты тут делаешь"? Мне ещё 10 минут ответ формулировать, а ты потом ещё до чего-то доебёшься, - вместо того чтоб получить на простой однозначный вопрос. Считай меня каким угодно долбоёбом, тред как раз для таких, я просто хочу узнать, равно ли численно g на горизонте событий скорости света. Или второй космической?

Аноним Вск 27 Апр 2014 01:28:49 #295 №110235 

Я правильно понимаю, что существование черных дыр не доказаны? Какие критерии необходимо узнать, чтобы можно было сказать точно - черные дыры существуют?

Аноним Вск 27 Апр 2014 02:23:07 #296 №110236 

>>110232
> вместо того чтоб получить на простой однозначный вопрос.
> я просто хочу узнать, равно ли численно g на горизонте событий скорости света.
Да ёб же тебя у ухо. Никаких эмоций не хватает. Ответь мне сперва на простой однозначный вопрос: равен ли численно здоровый пук сибиряка кубическому корню из ферментированной в желудке бомжа тертой с сахаром морковки? inb4: придираешься, занудствуешь!

sageАноним Вск 27 Апр 2014 02:24:46 #297 №110237 

>>110236
> в ухо!
эмоций не хватило

Аноним Вск 27 Апр 2014 02:35:24 #298 №110238 

>>110232
вторая космическая равна v=(2GM/R)^0.5
ускорение свободно падения g=GM/R^2
ты хочешь знать чему равно g на горизонте событий, так вот
v у нас равно c, а значит c=(2GM/R)^0.5;
(c^2)/2=GM/R
подставляем во вторую формулу
g=(c^2)/2R
то есть ускорение на горизонте событий очень зависит от радиуса, а радиус в свою очередь от массы. чем больше масса чд, тем дальше от центра находится горизонт событий и тем меньше ускорение свободного падения на нем.
в принципе, к сверхмассивным чд можно даже приблизиться не боясь быть разорванным приливными силами.
мимопроходил.

Аноним Вск 27 Апр 2014 05:10:12 #299 №110240 

>>110197
Как тебе реактор из РАО в кыштыме? ИЧСХ для совка это вполне естественно.

Аноним Вск 27 Апр 2014 05:17:50 #300 №110241 

>>110205
на горизонте численно равна скорости света скорость убегания. Не знаешь штаэта - гугли.
Ты такой тонкий траль, просто афигеть.
А на свободное падение тащемта похуй. Там свободно почти никогда ничего не падает, ибо большинство ЧД вращаются, окружены веществом и как следствие дисом аккреции.

Аноним Вск 27 Апр 2014 05:34:13 #301 №110242 

>>110235
>чтобы можно было сказать точно
нужно измерить радиус компактного объекта если он будет меньше или равен гравитационному радиусу - у нас есть ЧД.
До тех пор, вероятность того, что мы имеем дело с ЧД, имеет смысл мерить в сигмах. Однако уже сейчас для сокращения записей говорят о ЧД а не о кандидатах в ЧД

Аноним Вск 27 Апр 2014 07:38:43 #302 №110243 

>>110232
> Да ёпт, обязательно занудство это?
Да, ёпт, обязательно. Трудно ответить на вопрос, когда непонятно что ты вообще спрашиваешь.

Аноним Вск 27 Апр 2014 07:57:08 #303 №110244 

>>110230
>Ускорение свободного падения и скорость света.
Это примерно как сравнивать 2 килограмма картошки и 2 километра.

Аноним Вск 27 Апр 2014 11:12:27 #304 №110248 

У нас в тронотреде возник философский спор насчет планеты, на которой происходят действия из Игры Престолов. Если вы не знаете, то лето там может длиться 10 лет, а зима 100. Как такое возможно с вашей точки зрения?

Аноним Вск 27 Апр 2014 11:36:12 #305 №110251 
1398584172822.png

>>110248
Такая планета должна вращаться по вытянутой эллиптической орбите. Чем ближе к звезде, тем больше солнечной энергии она получает. В перицентре скорость планеты будет выше, и она будет проводить там меньше времени, это будет короткое лето. В апоцентре скорость меньше во много раз, и зима будет намного дольше.

Правда цикл все равно будет длиться один год этой планеты, единственное что можно тут предположить - что они зачем-то пользуются земными годами в качестве единиц измерения (ну неудобно им мерить своими годами в 110 лет каждый).

Насколько это возможно с точки зрения планетообразования - другой разговор. В нашей системе есть малые тела с сильно вытянутыми орбитами (Седна например), но полноценных планет нет. Может быть это и возможно. Возможность появления биологической жизни в нашем общепринятом смысле на этой планете тоже под сомнением, во всяком случае эволюционировать она должна была совершенно иначе, чем земная.

Другой вариант: в Игре Престолов не описывается вся планета ведь? Тогда все проще, они просто живут в дико холодном месте, а планета обычная земного типа с круговой орбитой и сменой времен года за счет наклонения, только вместо года совершает оборот за 110 наших лет. Правда, опять остается вопрос про годы как единицы измерения.

Еще один вариант - какая-нибудь вулканическая активность, циклический парниковый эффект, или еще какой-нибудь неастрономический фактор, вызывающий периодическую смену климата.

Аноним Вск 27 Апр 2014 11:37:10 #306 №110252 

>>110248
Планета на эллиптической орбите.
Двойная звезда.
Звезда переменной яркости.
Выбросы в атмосферу парниковых газов или частые извержения супервулкана.

Аноним Вск 27 Апр 2014 11:44:03 #307 №110253 

Но суть то не только в том, что именно всегда она длится 100 лет. Лето может длиться и 5 и 10 лет, и зима также. Это невозможно, да?

Аноним Вск 27 Апр 2014 11:47:17 #308 №110254 

>>110253
Возможно только если планета имеет сложную орбиту, взаимодействуя с несколькими другими телами. Даже не знаю, какой должна быть орбита такой планеты, чтобы быть при этом ещё и стабильной.

А так да, остаются только всякие вулканы с парниковыми эффектами, и
> Звезда переменной яркости

Аноним Вск 27 Апр 2014 12:21:43 #309 №110258 

>>110253
>Это невозможно, да?
Да хуй знает же. Нужны долговременные наблюдения, а там у них люди в живых долго не задерживаются.

Аноним Вск 27 Апр 2014 12:54:41 #310 №110261 

>>110251
>Возможность появления биологической жизни в нашем общепринятом смысле на этой планете тоже под сомнением, во всяком случае эволюционировать она должна была совершенно иначе, чем земная
Возможно там 500-1000 лет назад орбиту планеты, которая была подобной земной, искривила какая-нибудь мимопланета/звезда, а "старыми" годами пользуются по инерции.

Аноним Вск 27 Апр 2014 13:19:09 #311 №110263 

>>110244
Ну именно для света это похуй. Если g численно выше c, свет не сможет улететь. Если ниже - просто покраснеет. Или не так, и тут важна именно вторая космическая?

Аноним Вск 27 Апр 2014 15:19:49 #312 №110268 
1398597589727.png

>>110263
>тут важна именно вторая космическая
Это.
>Ну именно для света это похуй.
Свет не ускоряется. Свету похуй на g. Он сразу, мгновенно начинает ебашить со скоростью света.
>Если g численно выше c
Да у вас же фиксация, батенька.

А известно ли вам, что ускоряться можно меньше одной секунды? например бесконечно малую долю секунды И тогда даже при численном равенстве, достигнутая скорость будет меньше с?

Аноним Вск 27 Апр 2014 18:40:09 #313 №110281 

>>110228
Бамп реквесту

Аноним Вск 27 Апр 2014 19:54:59 #314 №110291 

>>108409
Сколько стоит космический туризм? Хочу свою тян на МКС захуйнуть. Ну и себя с ней тоже.

Аноним Вск 27 Апр 2014 19:56:27 #315 №110292 

>>108409
Сколько стоит космический туризм? Хочу свою тян на МКС захуйнуть. Ну и себя с ней тоже.

Аноним Вск 27 Апр 2014 20:32:38 #316 №110300 

>>110281
http://militera.lib.ru/explo/chertok_be/index.html

например

Аноним Вск 27 Апр 2014 21:56:00 #317 №110307 

>>110300
А есть что-то более подробно именно про королева? Где жил, где учился, куда ездил и т.п.? Или проще сразу гуглить его сухую биографию?

Аноним Вск 27 Апр 2014 21:59:59 #318 №110308 

>>110292
20000000$ на человека

Аноним Пнд 28 Апр 2014 00:25:02 #319 №110316 

>>110308
Можно просто совершить суборбитальный полет, это на порядки дешевле.

Аноним Пнд 28 Апр 2014 01:38:30 #320 №110317 

>>110307
Он же жил и учился в сознательном возрасте уже в СССР - там все у всех одинаково было, нечего читать особенно. В 30-е пускал с товарищами ракетки в вузике, но тогда они нахуй никому не нужны были, поэтому спонсирования никакого не было. Потом репрессии, шаражка в ВОВ, затем пришел к успеху.

Аноним Пнд 28 Апр 2014 07:34:16 #321 №110320 

>>110240
Но там же обычный разогрев был.

Атомного взрыва тред Аноним Пнд 28 Апр 2014 09:58:56 #322 №110325 

Короче я тут долго думал и вот до чего додумал: атомный взрыв произойдет при расщиплении любого атома или нет.Вот например если расщипить атом золота взрыв произойдет?

Аноним Пнд 28 Апр 2014 10:46:36 #323 №110326 

>>110325
При расщеплении ядра любого элемента выделяется энергия.

Проблема в том, что ядро атома золота очень стабильно, для того, что бы его расщепить, тебе понадобиться очень дохуя энергии. Это как надувать гондон промышленным компрессором. Гондон, безусловно лопнет и издаст свое "бум", но сквозь рев компрессора ты его не услышишь.


Аноним Пнд 28 Апр 2014 23:18:05 #324 №110348 

>>110206
>Мне шаблон рвёт то, что сверхновая получается из выгоревшей звезды, у которой по сути и энергия-то вся выработана.
У сверхновой реакции синтеза не способны компенсировать довление - она коллапсирует и потом разлетается, ибо коллапс дохуя энергии в себе несет - неслабо же, несколько масс солнц в единый миг обрушиваются к центру.

Аноним Пнд 28 Апр 2014 23:22:12 #325 №110350 

Откуда взялись элементы тяжелей железа в солнечной системе? Солнце в предыдущей итерации ведь прогорело только до железа а потом разлетелось.

Аноним Пнд 28 Апр 2014 23:25:00 #326 №110351 

>>110350
Солнце не прогорало, оно образовалось из остатков звезды первого поколения. Из них же и планеты вокруг солнца. А вот в остатках звезды первого поколения запросто и железо, и уран и прочая тяжесть.

Аноним Пнд 28 Апр 2014 23:41:46 #327 №110352 

>>110351
>оно образовалось из остатков звезды первого поколения
То есть того же самого Солнца? Оно горело, горело а потом PIZDANOOLO. Так откуда в остатках предыдущего Солнца взялись тяжелые элементы, да ещё в таком огромном количестве.

Аноним Втр 29 Апр 2014 00:14:11 #328 №110359 

>>110352
Из вообще другой звезды. Она собрала в себе огромное количество первичной материи (водород с гелием) погорело считанные миллионы лет и ёбнуло, расшвыряв еще кучу не прореагировавшего водорода и гелия, из которых образовалось Солнце (и еще много других звезд), и еще всякого барахла, из которого преимущественно состоят планеты (не только в нашей Солнечной системе). А от той сверхновой осталась черная дыра. Где она - хуй знает.

Аноним Втр 29 Апр 2014 00:14:43 #329 №110360 
1398716083890.jpg

>>110352
И сюда же второй вопрос - куда делись планеты после взрыва звезды первого поколения? Из них собрались наши нынешние планеты, или они все ещё болтаются где то за границами Солнечной системы или вообще улетели в межзвездное пространство?

Аноним Втр 29 Апр 2014 00:17:54 #330 №110361 

>>110359
Но тогда все это барахло раскидало бы вообще по всему космосу, он же бесконечен. Как оно снова собралось в кучу - ведь любой газ стремится занять все доступное пространство

Аноним Втр 29 Апр 2014 00:31:07 #331 №110362 

>>110360
Летают где-нибудь.
>>110361
Гравитация, сэр. Элементов тяжелее не то что железа, но и гелия в Солнечной системе, если считать по массе, очень даже немного. Того что тяжелее железа так вообще совсем чуть-чуть, спасибо хотя бы за это неопределенному количеству сверхновых миллиарды лет назад.

Аноним Втр 29 Апр 2014 01:00:02 #332 №110366 

>>110360
Планет в первом поколении не могло быть. Могли быть водородные газовые гиганты - и может быть они бороздят просторы вселенной сейчас.
Почитай википедию - там оче доступно все описано. И про эволюцию и поколения звезд, и про образование планет. Все.
Можно дискавери на досуге глянуть.
У тебя прямо космология для дошкольников получается.

Аноним Втр 29 Апр 2014 07:20:39 #333 №110373 

>>110348
> она коллапсирует и потом разлетается, ибо коллапс дохуя энергии в себе несет - неслабо же, несколько масс солнц в единый миг обрушиваются к центру
Из-за чего разлетается-то? Несколько масс солнца обрушиваются под действием сил гравитации. Ну и обрушились они, пусть так и лежат обрушенные. С чего им разлетаться? Гравитация отключается что ли?

Аноним Втр 29 Апр 2014 09:31:04 #334 №110376 

>>110317
В том-то и дело, что хочется очень подробно прочитать про его учебу, эти ракетки в вузе с товарищами и т.п.

Аноним Втр 29 Апр 2014 10:16:18 #335 №110379 

>>110373
>Ну и обрушились они, пусть так и лежат обрушенные. С чего им разлетаться?
Это не задрипанная пятиэтажка, которая обрушившись будет лежать, это, мать его, сотни тысяч масс всей нашей планеты, и падая, не удерживаемые давлением термояда, они выделяют дохулиард энергии.

Аноним Втр 29 Апр 2014 10:34:18 #336 №110380 

>>110373
Огромное давление создается, и, что называется, последняя вспышка. И очень даже мощная.

Аноним Втр 29 Апр 2014 10:45:09 #337 №110381 

С какой скосротью идёт время для земного наблюдателя в далёких галактиках? Ну то есть они удалляются от нас - не влияет ли это на видимый ход времени?

Аноним Втр 29 Апр 2014 10:48:52 #338 №110383 

>>110373
Ну там ещё в волне сжатия должны термоядерные реакции происходить, но вообще почему сверхновая 2 типа взрывается - ещё открытый вопрос. По всем расчётам нихуя не должна. Вроде грят, нейтрино наконец-то проявляют участие а не только сотни энергии пиздят, суки

Аноним Втр 29 Апр 2014 11:00:17 #339 №110384 

>>110379
При падении энергия выделяется? Энергия же уходит на преодоление межядерных сил.

>>110380
А вспышка отчего? Она ведь тоже удерживается гравитацией.

Аноним Втр 29 Апр 2014 11:22:08 #340 №110386 

>>110373
Сверхновая - это не пятиэтажка как верно подметил один из ораторов. Но даже при падеже пятиэтажки пыль поднимается столбом. Не говоря уж о небоскр ёбах.
Гравитационный коллапс - ступенчатый процесс, плохо изученный и разные модели показывают разные результаты.
Так вот то, что мы видим при вспышке - не более чем пыль, хотя ее относительно много. В компактный объект обращается только ядро звезды. Во внешних же оболочках могут успевать происходить не только водород-водородные, но даже железо-железные реакции.

Аноним Втр 29 Апр 2014 17:52:50 #341 №110402 

А кстати, пацандрэ, если нейтрино имеют массу (пусть и очень малую), то они должны вступать в гравитационное взаимодействие, так? А что если они собираются в нейтринные "звезды"? Они же так могут и сингулярность образовать. Кстати, и к обычной материи они притягиваются тогда, и образуют сгустки в центрах звезд. Что думаете? Есть ли какие-нибудь гипотезы об участии нейтрино в наблюдаемых или предполагаемых космических явлениях (кроме скрытой массы).

Аноним Втр 29 Апр 2014 18:19:41 #342 №110403 

>>110402
Нейтринный холодильник.
А звезды - слишком легкие чтобы привлечь внимание нейтрин. Только нейтроныые звезды достаточно плотны чтобы задерживать их. Но они мелкие, а нейтрины - слишком разрежены.

Аноним Втр 29 Апр 2014 19:01:16 #343 №110408 
1398783676441.png

Каков будет удельный импульс у ТРД для полета в бескислородной атмосфере, в котором в камеру сгорания, помимо топлива, впрыскивается окислитель?

Аноним Втр 29 Апр 2014 19:23:45 #344 №110413 

>>110408
Никакой. Взорвётся.
Под окислитель нужно переделывать, в итоге на выходе будет ЖРД.

Аноним Втр 29 Апр 2014 21:27:00 #345 №110420 

>>110408
Смотря какой окислитель, смотря какое топливо.

>>110413
Хранить атмосферный воздух в баках, например, и подавать как и при обычном полете бессмысленно и беспощадно, но ведь можно жи

Аноним Втр 29 Апр 2014 21:30:37 #346 №110423 

>>110408
>Каков
Да хрен его знает.
>в бескислородной атмосфере
Если атмосфера близка к нейтральной - нивзлитит.
А вот если атмосфера - восстановительная, Юпитер там, сатурн например, стоит изучить возможности построения двигателя, который возит бак с окислителем с собой жидкий фтор, алало, а топливо забирает из атмосферы.

Аноним Втр 29 Апр 2014 21:32:44 #347 №110424 

>>110206
После коллапса центрального ядра к центру звезды быстро притягивается внешняя оболочкаочень-очень быстро. Потом вещество оболочки отскакивает от ядра и образуется ударная волна, которая инициирует термоядерные реакции. Этого хватает для сброса оболочки сверхновой с колоссальной скоростью. И еще ударную волну подпитывают нейтрино из ядра, так что волна нехилая совсем

Аноним Втр 29 Апр 2014 23:18:25 #348 №110429 
1398799105695.jpg

>>108409
Космач, я не узнаю тебя, что происходит? Столько вопросов на которые интересно отвечать.

Ты обдолбался?

Аноним Срд 30 Апр 2014 00:39:17 #349 №110431 

>>110384
>При падении энергия выделяется? Энергия же уходит на преодоление межядерных сил.
Еще как выделяется. Как говорил Рэндэлл Монро в 73 выпуске What-if, The physicist who mentioned this problem to me told me his rule of thumb for estimating supernova-related numbers: However big you think supernovae are, they're bigger than that. - какими бы большими ты ни представлял сверхновые - они еще больше.
Про то, как же мы представляем процессы в сверхновых достаточно популярно описано в http://ru.wikipedia.org/wiki/Сверхновая_звезда
Ты проявляешь достаточно любознательности, чтобы быть в состоянии самостоятельно ознакомиться с этой небольшой статьей.

Аноним Срд 30 Апр 2014 03:23:35 #350 №110434 

>>110429
Что ты как барышня, увидевшая хуй.
Давай без этих обмороков: что конкретно тебя удивляет\тревожит\беспокоит\не устраивает?

Аноним Срд 30 Апр 2014 03:32:25 #351 №110436 

>>110402
Или ещё вариант: нейтрино собираются вокруг горизонта событий, чинят там угар и содомию шопиздец.

Аноним Срд 30 Апр 2014 06:58:01 #352 №110438 

>>110424
> вещество оболочки отскакивает от ядра
Как каучуковый мячик?

>>110431
Статью читал, и, например, отскок вызывает удивление.

Аноним Срд 30 Апр 2014 08:51:38 #353 №110440 

>>110438
если М(масса звезды)<1,4Мс(массы Солнца), то взрыва не происходит вообще.Если же 1,4Мс<M<2Mc, то гравитации не хватает, чтобы удержать вещество на оболочке и избежать отскока. Если М>2Mc, то вещество уже не может никуда отскочить и процесс сжатия необратим, образуется черная дыра

Аноним Срд 30 Апр 2014 10:26:48 #354 №110443 
1398839208339.gif

>>110440
Ну что ты несешь. Пруфани свои утверждения, что ли. Всего лишь при 2М звезды не сбрасывают оболочку при коллапсе, офигеть.

Аноним Срд 30 Апр 2014 11:08:57 #355 №110444 

Спейсач, подскажи аргументов за реальность полетов на Луну. Шеф уперся рогом, мол пиздят все, кина снимать больше не хотят. И ведро конспирологии вывалил. Как мне, обычному дивану, вразумить шефа, параноидального дивана?

Аноним Срд 30 Апр 2014 11:39:30 #356 №110446 

>>110444
Есть же полноценная статья на вики - http://ru.wikipedia.org/wiki/Лунный_заговор. Если аргументы оттуда его не убедят, то он слишком глубоко погряз в войне с жидорептилоидами.

Аноним Срд 30 Апр 2014 12:02:25 #357 №110448 

>>110446
Щепоть реголита тебе.

Аноним Срд 30 Апр 2014 12:15:39 #358 №110449 

>>110446
Статья, на самом деле, так себе.
Эта получше http://www.skeptik.net/conspir/moonhoax.htm

Аноним Срд 30 Апр 2014 13:22:36 #359 №110453 

Как орлы стартанули с Луны и стыковались коламбием? Это наверно очень сложно.

Аноним Срд 30 Апр 2014 14:08:23 #360 №110455 
1398852503155.jpg

>>110453
Вот например более-менее детальное описание процедуры взлета и стыковки.
http://history.nasa.gov/afj/loressay.htm
Жаль не настолько в деталях как с посадкой, а ведь это даже сложнее было
http://spaceref.com/missions-and-programs/nasa/apollo/apollo-lunar-landing-mission-symposium/apollo-lunar-module-landing-strategy.html

Кстати все использованные взлётные ступени ебанулись назад об луну, орбита деградировала.

Аноним Срд 30 Апр 2014 15:41:38 #361 №110457 

>>110443
речь шла о сверхновых 1-го типа, а не второго

Аноним Срд 30 Апр 2014 15:48:25 #362 №110458 

>>110443
думаешь двух масс Солнца не хватит для сверхновой 1 типа? пффф

Аноним Срд 30 Апр 2014 16:36:50 #363 №110459 
1398861410893.jpg

Почему фигней в Кресте Эйнштейна именно четыре? Разве не должно получаться что-то вроде кольца?

Аноним Срд 30 Апр 2014 16:38:37 #364 №110460 

>>110458
Если облако Оорта разом скинуть в Солнце, заведется?

Аноним Срд 30 Апр 2014 17:07:28 #365 №110464 

>>110460
ну у Солнца топлива еще на 5 млрд лет в любом случае

Аноним Срд 30 Апр 2014 17:55:31 #366 №110471 

>>110464
Но я не хочу спасать человечество, муахаха! Шутка, конечно хочу. Может придется экстренно покинуть это место, а люди не захотят например вылезать из уютной колыбельки. А тут раз - облако Оорта падает на Солнце, кто не построил межзвездный ковчег - я не виноват!

Аноним Срд 30 Апр 2014 18:44:20 #367 №110477 

>>110465
Все от массы зависит. Если масса звезды большая, то после окончания топлива она сжимается до черной дыры, а если маленькая, то гравитации не хватает для такого сжатия, она сбрасывает внешнюю оболочку и остается голое ядро, коим и является белый карлик. Научпоп конечно, но будь ты физиком то не спрашивал бы а таком здесь.

Аноним Срд 30 Апр 2014 18:47:22 #368 №110478 

>>110465
И еще. Пульсар - это не начальная стадия черной дыры. Пульсар это нейтронная звезда, полюсное излучение которой, во время вращения, проходит через Землю.

Аноним Срд 30 Апр 2014 23:27:18 #369 №110504 
1398886038533.jpg

Спейсач, я сам гуманитарий, но неожиданно заинтересовал вопрос за магнитосферу. Запостил в вопрос в тредж любителей альтернативной истории, но вангую там совсем поехавших наблюдателей НЛО.
Суть вопроса следующая: в 70х мурриканцы несколько раз слетали туда-назад на луну, обмотавшись тефлоновой фольгой и у них все прошло замечательно.
В 86 году в СССР ебанул чернобыль и в разборах радиоактивной пылитм принимали участие разнообразные роботы, как советские так и не очень, и по словам из обучающих фильмов (я гуманитарий еще раз) электронные мозги у них кипели только в путь.
Как тогда получилось, что СССР не догадался обмотать роботов и добровольцев той же легкой тефлоновой фольгой, а мурриканцы пролетели магнитосферу, да еще и обратно вернулись за почти 17 лет до события?
Отнеситесь с понимаем, пожалуйста. Вы же знаете, у всех это бывает.

Аноним Срд 30 Апр 2014 23:59:04 #370 №110505 
1398887944042.jpg

>>110504
В Чернобыле для людей было опасно в первую очередь интенсивное гамма-излучение и попадание радиоактивных изотопов внутрь (ну нейтронное облучение тоже не сахар), ибо от альфа- и бета-излучений (которых там тоже было дохуя) относительно спасает костюм химзащиты (как и от дыхания цезиево-плутониевой пылью, например, да). В магнитосфере же структура излучения совершенно другая, защищались от него, кстати, не тефлоновой фольгой (что это?), а довольно толстыми алюминиевыми стенками. Кроме того, американцы летали через радиационные пояса быстро-быстро (буквально пару часов, емнип), так что астронавты большой дозы не получали.
Что же до воздействия радиации на электронные мозги техники - это отдельная и очень длинная тема, на хабре почитай. Но в целом - в радиационные пояса спутники и сейчас стараются особенно не отправлять без необходимости.
Грубо говоря - хочешь защититься от радиации - замуруй себя в герметичную железобетонную капсулу со стенками в пару метров, иначе никак.
Сумбурно как-то получилось, но уж очень вопрос такой.

Аноним Чтв 01 Май 2014 00:32:19 #371 №110510 

>>110505
Я немного погуглил и понял, что МэВ это вообще не очень вменяемый показатель. Т.е. их может быть много, но они не опасные, правильный показатель - милизиверты.
У нас есть какие-то нормальные подтвержденные табличные данные о том, сколько в этих поясах этих самых милизивертов в тушку прилетает? Кроме данных мистера кьюриосити.

Аноним Чтв 01 Май 2014 00:33:28 #372 №110511 
1398890008631.gif

>>110510
Впрочем, я скорее всего сморозил чушь. Грустно быть гуманитарием.

Аноним Чтв 01 Май 2014 00:37:58 #373 №110512 

>>110505
Тогда надо начать с того, как именно поражает лучевая болезнь.
Вот смотри, по чернобылю я нашел тут: http://radscler.narod.ru/doses/chernobyl_doses.html
>У таких изотопов как Йод-132, Лантан-140, Празеодим-144, Рутений-106, Иттрий-90 максимальная энергия бета-частиц превышает два МегаэлектронВольта (МэВ), что позволяет им пролетать в воздухе более 10 метров, а в биологической ткани до 1,5 см.
Т.е. опасность радиации именно в том, что всякие не сильно полезные для здоровья частицы, аки пули, проникают в нашу тушку и там остаются?
Я что-то совсем запутался.

Аноним Чтв 01 Май 2014 00:44:10 #374 №110513 

>>110512
Да, похоже это логично.
Значит для прямого сравнения чернобыля и магнитосферы с поледующим "быстро-быстро" мне надо въехать в разницу между составом выброшенных альфа-бета-гамма частиц в чернобыле и такими же в этой магнитосфере, и привести их к некому общему базису, я правильно понимаю?

Аноним Чтв 01 Май 2014 01:14:44 #375 №110515 

>>110513
Я нагуглил кучу материалов, всем спасибо, прочитаю и выберу победителя по кол-ву красивых формул и искрометного текста.

Аноним Чтв 01 Май 2014 03:25:26 #376 №110519 
1398900326167.jpg

>>110515
И не забудь про разницу между внешним и внутренним облучением. В случае гамма-квантов практически похуй, в случае же альфа-частиц разница колоссальна.
Суть не в том, что они остаются (эта проблема с теми частицами, которые еще не распались, вроде няшных йода-132, стронция-90 и цезия-137), а в том, что они несут с собой нехилую энергию и идя через организм - производят пиздец, весьма разнообразный и многоступенчатый, но в основном - если имеется в виду именно лучевая болезнь - через перекисное цепное окисление липидов.
курс радиационной биофизики лучше почитай, попробуй, еще захочешь

Аноним Чтв 01 Май 2014 03:28:08 #377 №110520 
1398900488940.gif

>>110518
Субраманьян Чандрасекар как бы определяет верхний предел массы белого карлика в 1.44 солнечных.

Аноним Чтв 01 Май 2014 03:30:27 #378 №110521 
1398900627393.jpg

>>110520
а, ну да - если интересует уже черная дыра, то обращайся к пределу Оппенгеймера-Волкова

Аноним Чтв 01 Май 2014 07:01:34 #379 №110524 

>>110455
Жаль что всё на английском. Это действительно самый опасный участок миссии.

Есть ли где наглядные симуляции миссии. В целестии или еще где?

Аноним Чтв 01 Май 2014 11:17:40 #380 №110525 

>>110518
предел Оппенгеймера-Волкова

Аноним Чтв 01 Май 2014 11:31:26 #381 №110527 

>>110525
Мой предел Оппенгеймера-Волкова лучше твоего.

Аноним Птн 02 Май 2014 09:38:22 #382 №110572 

>>110521 соус пикчи?

Аноним Пнд 05 Май 2014 07:10:32 #383 №110752 
1399259432103.gif

>>110504
>>110505
Вообще-то суть не в скорости пролёта поясов, а в траектории: аппарат летел в плоскости, перпендикулярной плоскости орбиты Луны (а не вдоль).

>>110512
> аки пули, проникают в нашу тушку и там остаются?
Вред наносят частицы, испускаемые при распаде нестабильных изотопов. При естественном распаде, в зависимости от изотопа, это электроны (бета), ядра гелия (альфа) или гамма (электромагнитное излучение). Они "аки пули, проникают в нашу тушку" и всё. То, что они там остаются, это хуита, их очень мало. Важно то, что эти "пули" при движении через вещество разрушают его, оставляют после себя след распидорашенных молекул.
По поводу внутреннего и внешнего облучения подразумевается расположение изотопов. Изотопы, это "заряженные пистолеты", готовые выстрелить в любой момент.
Если изотопы находятся вне организма, то практически похуй на альфа- и бета-излучающие изотопы, потому что такое излучение даже до кожи не дойдёт. На гамма не похуй, но важно то, что даже если испачкаешь руки, то загрязнение элементарно смывается.
Если изотопы попадают внутрь организма, то они, как химические элементы, захватываются в обмен веществ, распространяются по организму, оседают в определённых органах (как, например, стронций, аналог кальция, оседает в костях). После этого изотопы крайне трудно выводить из организма, а тем временем эти "пистолеты" сидят там и пуляют во все стороны, естественно, живые ткани ловят "маслины".

Про состав радиационных поясов сказано, что в основном там электроны и протоны. Причём у протонов десятки МэВ, а у электронов всего сотня кэВ. Такие электроны даже картон с трудом пройдут, а тормозное гамма от них легко гасится сантиметром алюминия. Протоны покруче будут, они в приоритете, от них нужно защищаться материалом с наибольшей плотностью и наименьшим атомным номером, под это дело прекрасно подходит алюминий, который, собственно, и применяется в авиации и космосе. И метровой толщины стенок тоже не нужно. Протоны быстро мрут в нескольких сантиметрах алюминия.

Проводить параллели с Чернобылем нельзя. Ситуации отличаются кардинально, не только видом излучений, их интенсивностью и спектром, но и видом опасности. В космосе мы имеем дело с готовыми "пулями" (защита от них - одна задача), в Чернобыле - не только с "пулями", но и с "пистолетами" (защита от которых - ещё одна нелёгкая задача).

Аноним Пнд 05 Май 2014 12:29:30 #384 №110759 
1399278570627.jpg

Спейсач, приснился охуенный сон, спешу поделиться.
Значит так, человечество, составив с помощью самореплицирующихся дронов-автоматов "карту" Галактики, приступило к осуществлению колоссального астроинженерного проекта. Искусственным образом давали подходящим звёздам и планетам большой импульс силы и "сгоняли" их в колоссальный кластер, который в итоге стал в себя включать миллионы обитаемых миров. Однако при этом диаметр этого кластера не превышал нескольких парсек, так и были решены проблемы дальних межзвёздных перелётов. Также были организованы трассы ионизированного газа, которые регулировали напряжённость магнитного поля в кластере, а на подхвате - тысячи других звёзд, которые можно использовать как "консервы".
Как можно изменить импульс звезды, каким-нибудь искусственно организованным джетом?
Есть ли фантастические проекты такого или большего масштаба в литературе?

Аноним Пнд 05 Май 2014 13:09:09 #385 №110760 

>>110759
> Как можно изменить импульс звезды, каким-нибудь искусственно организованным джетом?
Без реактивного движения пока никак. Закон сохранения импульса всему виной.

Аноним Пнд 05 Май 2014 13:19:40 #386 №110763 
1399281580356.jpg

>>110759
Пиздец ты наркоман.
> Как можно изменить импульс звезды, каким-нибудь искусственно организованным джетом?
Джет - это полярный выброс. Хинт: полюсов два, т.е. работает он одинаково в обе стороны. А так-то нужно собрать аккреционный диск, всего ничего!

Аноним Пнд 05 Май 2014 13:29:31 #387 №110764 

>>110763
Я условно назвал это "джетом", имелся в виду просто мощный выброс вещества, т.е. >>110760. Вопрос в том, как создать тягу.
Нашёл вот ещё что
http://ru.wikipedia.org/wiki/Звёздная_машина

Аноним Пнд 05 Май 2014 14:57:21 #388 №110765 

>>110759
> Могут двигать звезды
> Не могут в межзвездные путешествия

Аноним Пнд 05 Май 2014 15:46:53 #389 №110768 

>>110408
>Каков будет удельный импульс у ТРД для полета в бескислородной атмосфере, в котором в камеру сгорания, помимо топлива, впрыскивается окислитель?
Поезд из точки А в точку Б идёт со.... бла-бла-бла проводник выпивает 2 стакана чая в час.... бла-бла-бла сколько мне лет?
28, Марь Ивановна.
Правильно. Как посчитал, Вовочка?
Мне 14, и отец меня полудурком зовёт.

>удельный импульс у ТРД
Ну ты понял. Дельта ви не является характеристикой ТРД. Здесь вам не KSP
А так- взлетит. Нагрев рабочего тела есть? Есть. А за счёт чего-строго похуй, хоть ТЭНами грей, хоть реактором.
Пиндосы делали проект прямоточного двигателя с ядерным реактором. http://ru.wikipedia.org/wiki/Плутон_(проект)
Не совсем то, но похуй, пойдёт.

Аноним Пнд 05 Май 2014 16:08:21 #390 №110769 

>>110765
"Толкание" звёзд законам физики не противоречит, хотя и сложноосуществимо; перемещение со скоростью, превышающей с законам физики противоречит. Альтернатива - многотысячелетние перелёты.

Аноним Пнд 05 Май 2014 16:09:42 #391 №110770 

>>110768
>Здесь вам не KSP
Там тоже не.

Аноним Пнд 05 Май 2014 21:15:31 #392 №110789 

>>110759
но зведы к месту назначения ведь тоже будут перемещаться туеву хучу лет

Аноним Пнд 05 Май 2014 22:32:21 #393 №110797 

>>110769
Мы уже в пути, если ты не заметил.

Аноним Пнд 05 Май 2014 22:44:28 #394 №110798 

>>110789
Ну, несколько сотен миллионов лет потерпеть, зато потом последующие миллиарды лет можно будет к бабушке в систему Ригеля всего за месяц долетать.
>>110797
На неправильном пути. Не хотеть бестолково вращаться вокруг центра Галактики с периодом в 200 миллионов лет.

Аноним Втр 06 Май 2014 02:10:12 #395 №110806 

>>110798
>>110798
>На неправильном пути
А какой правильный, стесняюсь спросить.
Около 11 800 г. ближайшей к земляшке звездой после слонца, станет звезда Барнарда 3.8св.г, например.
Потом еще какая-нибудь. Во времена динозавров, Проксимой тоже не αCentauriC была. информация отсутствует

Аноним Втр 06 Май 2014 02:12:31 #396 №110807 

>>110798
>к бабушке в систему Ригеля
Да ну нахуй. К тому времени как слонце туда долетит, у бабушки пердак ригель бомбанет как сверхуновая.

Не хотеть.

Аноним Втр 06 Май 2014 13:43:46 #397 №110822 

Почему космонафты в открытом космосе не дохнут от радиации?

Аноним Втр 06 Май 2014 14:17:33 #398 №110823 

>>110822
Не успевают.

К тому же открытый космос открытому космосу - рознь.

Есть низкая орбита при спокойном солнце, а есть орбита за пределами магнитосферы, да подкоронарным выбросом солнцаю

Аноним Втр 06 Май 2014 14:18:58 #399 №110824 

>>110823
*на пути коронарного выброса

Аноним Втр 06 Май 2014 16:43:26 #400 №110826 

>>110769
> перемещение со скоростью, превышающей с законам физики противоречит
Не противоречит даже известным.

Аноним Втр 06 Май 2014 17:50:47 #401 №110832 
  1. До какой температуры максимум можно нагреть Луну чтобы она осталась одним целым телом?
    2. Сравнится ли разогретая Луна с Солнцем по светимости и потоку тепла. Как это отразится на климате Земли?
Аноним Втр 06 Май 2014 18:29:17 #402 №110837 

>>110832
Ты ебанутый?

Аноним Втр 06 Май 2014 21:07:31 #403 №110859 

>>110837
> тред тупых вопросов

>>110832
1. А с чего бы ее должно распидорасить при нагревании? Она и так жидкая (в планетарных масштабах), оттого и круглая - удерживает сама себя.

2. Смотря насколько разогретая. Если достаточно - всем пизда

Аноним Втр 06 Май 2014 21:56:01 #404 №110870 

>>110859
>2. Смотря насколько разогретая. Если достаточно - всем пизда

Поясни.

Аноним Втр 06 Май 2014 22:05:07 #405 №110871 

>>110870
2. Если разогреть Луну до миллиарда градусов, всем пизда.

1. Сравнится ли разогретая Луна с Солнцем, не сравнится ли - зависит только от температуры разогрева.
Ну и будет ли она остывать или не будет - зависит от накачки энергией. Будет накачка - будет изменение климата, не будет - ну что-то изменится, но заметят только потомки, например.

Аноним Втр 06 Май 2014 22:40:31 #406 №110873 

>>110837
>Ты ебанутый?
Да.
>>110859
>А с чего бы ее должно распидорасить при нагревании?
Если скорость движения молекул превысит вторую космическую для Луны - то распидорасит. А вот какой температуре она соответствует я не знаю, потому и спросил.
>>110871
> Если разогреть Луну до миллиарда градусов, всем пизда.
Не, ну понятно, что при миллиарде градусов она просто разлетится в виде расширяющегося облака плазмы и по пути сдует земную атмосферу. А как насчет промежуточных значений?
>Ну и будет ли она остывать или не будет - зависит от накачки энергией
Кстати да, если накачки энергией нет, то как быстро она будет остывать.
>Сравнится ли разогретая Луна с Солнцем, не сравнится ли - зависит только от температуры разогрева.
Ага, вот и я так думаю. Вот значит некая магическая сила разом нагрела всю Луну до предельной температуры, при которой она ещё держится единым куском а не стремится занять собой все доступное пространство. На небе мы увидим светящийся диск, фактически второе Солнце. Постепенно Луна будет остывать и светимость диска будет снижаться. под конец она полностью погаснет в видимом спектре, но ещё долгое время будет светить тепловым излучением.
Ну и сразу вопросец знатным косманам - после остывания планета станет идеально гладким сфероидом, или на ней появятся кратеры и неровности.

Аноним Втр 06 Май 2014 22:49:54 #407 №110874 
1399402194382.jpg

Собственно вопрос про раскаленную Луну возник у меня после прочтения этого:
Греческий философ Анаксагор говорил, что Солнце — это не колесница Гелиоса, как учила греческая мифология, а гигантский, «размерами больше, чем Пелопоннес», раскалённый металлический шар
И вот я представил себе что какой нибудь бог ради смеха организовал систему освещения планеты именно таким образом.

Аноним Втр 06 Май 2014 23:27:28 #408 №110875 
1399404448636.png

>>110873
>А вот какой температуре
приклейтед. Вторая комическая на луне 2.38 км/с, еси шо

Аноним Втр 06 Май 2014 23:42:16 #409 №110877 

>>110873
>идеально гладким сфероидом
Вот это вряд ли. Орбита не идеальная, но даже при идеальной будет яйцеобразность, приливные силы же. Обычное остывание через поверхность даст поначалу не столько кратеры, сколько епические трэщины, тектонические плиты, субдукцию, сейсмичность как на земляшке, вот это все.
Кратеры будут потом, вулканические. Не такое это уж быстро дело.

Ну и для луняшки это был бы заебатый повод завести свою собственную магнитосферу, с поясами ван-аллена и резонансами шумана

Аноним Срд 07 Май 2014 00:08:37 #410 №110880 

>>110875
Шо то я нихрена не понял, откуда брать недостающие значения.
>>110877
А примерно сколько будет остывать Луна? Тысячу лет, миллион или больше?

Аноним Срд 07 Май 2014 00:39:18 #411 №110882 

>>110880
>миллион
Скорее это и даже дольше.
Земля вон до сих пор остывает. Правда это таки из-за Луны. Собсна, луна до сих пор не остыла, у нее все еще есть небольшое расплавленное ядро.
>недостающие значения.
какие значения? универсальную газовую постоянную? молекулярную массу реголита? загугли.

Аноним Срд 07 Май 2014 23:08:53 #412 №110967 

Берем дроздофил и отправляем на марс в стеклянном куполе с воздухом и едой. Распидорасит их радиацией или нет?

Аноним Чтв 08 Май 2014 02:05:42 #413 №110990 

>>110967
Планируют пускать вместе со следующим марсоходом. Правда не дрозофил, а просто растения.
А так - хуй знает, какой-нибудь генетически модифицированный саксаул может и выжил, и размножился бы на дне долин Маринера.

Аноним Чтв 08 Май 2014 02:59:29 #414 №111000 
1399503569952.jpg

>>110967
>Распидорасит их радиацией или нет?
нет. если отправят овер 9000 то на выходе получим марсианских мух которые эволюционируют в марсианских НЕХ
Хотя по идее надо начинать с отгрузки РНК бочками

Аноним Чтв 08 Май 2014 03:46:21 #415 №111003 

>>110990
>генетически модифицированный саксаул может и выжил
Есть мнение, что обычную капусту можно высаживать в открытый грунт, если бы не адовый дубак.

Аноним Чтв 08 Май 2014 10:58:40 #416 №111018 
1399532320951.jpg

>>111003
Какой нахуй саксаул, какая капуста! Там же 1/100 атмосферного давления, нет жидкой воды, холод и 1/3 солнечного света от земного. Проще капусту в открытом грунте на леднике Росса выращивать чем в таких адовых условиях.

Аноним Чтв 08 Май 2014 12:18:41 #417 №111021 

>>111018
Для того Марс и предусмотрел днище долин Маринера с атмосферным давлением повыше (а парциальное давление углекислого газа там охуеть как выше, чем на Земле), низкими перепадами температур и теоретически возможным существованием жидкой воды в определенном диапазоне температур. А про солнечный свет ты какому-нибудь мху в еловом лесу расскажи, он и от 1/3 сдохнет.

Аноним Чтв 08 Май 2014 17:56:07 #418 №111042 
1399557367148.jpg

>>111021
>а парциальное давление углекислого газа там охуеть как выше, чем на Земле
0.004 на Земле, а на Марсе 0.01. На Земле это кстати идеально для растений, в теплицах такое делают.
>существованием жидкой воды в определенном диапазоне температур
Ягель! Промерзает насквозь на вечной мерзлоте, полярная ночь по полгода, но выживает и весной опять растет. Вообще если его прям сейчас туда засадить, он сцуко там будет жить! Не хочется только местную гипотетическую флору упустить.

Аноним Птн 09 Май 2014 17:25:16 #419 №111115 

>>108409
Как тебе идея анон?
Организуется отдельное независимое гос-во, условно Космопорт которое в первое время спонсируется несколькими странами, все заинтересованные страны отправляют туда своих ученых, астронавтов, космонавтов ect. Между всеми страна заключается договор о нейтралитете этой страны. Главная цель Космопорта исследованные и колонизация космоса в инетресах Земли. Правитель - совет учены. Любые действия против науки - депортация. Нет влияния внешних сил.

Аноним Птн 09 Май 2014 20:43:21 #420 №111144 

>>111115
Идея отдает наивностью и максимализмом. Не учтено множество факторов. Одним из основных негативных факторов для реализации подобной идеи является низкий уровень консолидации отдельных государств на планете.

Аноним Птн 09 Май 2014 21:24:54 #421 №111154 

Что любовнее? Стэнфордский тор, Цилиндр О"Нейла или Сфера Барнала? Желательно с объяснением.

Аноним Птн 09 Май 2014 22:31:57 #422 №111157 

>>111144
>Одним из основных негативных факторов для реализации подобной идеи является низкий уровень консолидации отдельных государств на планете.

Однако скинуться на проекты вроде МКС и ИТЕР ума хватило.
Люди всегда обьединяются или для чего-то, или против кого-то.
Если высказанный >>111115 проект будет сулить профит который пересилит жабу, тогда его реализуют.

Аноним Птн 09 Май 2014 22:47:11 #423 №111158 

>>111154
Тор лучше. Он прочней, ремонтопригодней и более рационален.

Аноним Суб 10 Май 2014 05:40:27 #424 №111169 

>>111154
От размеров же зависит. Для километров - тор, для десятков-сотен километров цилиндр, для десятков миллионов километров сфера со звездой в центре

Аноним Суб 10 Май 2014 10:52:50 #425 №111178 

>>111157
>профит который пересилит жабу
Ты не пони.

Такие проекты могут осуществляться только при диктатуре. Может и не при гитлеровской, а какой-то помяхше, но все же железной рукой. Но как только в какой-то отдельно взятой стране появляется такая ручечка, все остальные начинают ее демократить.

Это т.н. "ведро крабов".

Мкс - всё, свою основную функцию выполнила и теперь как мертвый боров.
Бак - тоже всё, хотя статистику еще нарабатывает, однако для бОльшей светимости пучка уже нужен более мощный ускоритель.
Итэр - уже пиздецки дорогой, но все еще экспериментальный в то время как нужен по крайней мере, ДЕМО.

Отсутствие жабы - как химическая ракета, шумно пыльно но быстро. Вот проекты новых чехлов на кикстартере и плодятся.
Профит от науки как плазменный двигатель, отличные характеристики, но работают только в сферическом политическом вакууме, которого просто нет.

Аноним Суб 10 Май 2014 13:35:55 #426 №111184 

>>111178
>Такие проекты могут осуществляться только при диктатуре.
Вообще-то все современные "демократии" это просто ширма, если на то пошло.
А про диктатуру-напомню про феерического Хрущёва, который и кибернетику лженаукой назвал, и генетика у него - "продажная девка империализма" и художники-авангардисты - "пидорасы" . Науку во все времена продвигает наличие талантливых людей и нормальное финансирование. Давай spc не буде м в политач превращать, Ок?

>Это т.н. "ведро крабов".
Что? Про банку с пауками слышал, про клетку с обезьянам тоже.

>Мкс - всё, свою основную функцию выполнила и теперь как мертвый боров.
Поэтому и написал, что будет МКС-2. С большей автономией, отработкой длительного полёта, вплоть до загрузки раз в 2-3года с полной автономией.

>Бак - тоже всё, хотя статистику еще нарабатывает, однако для бОльшей светимости пучка уже нужен более мощный ускоритель.
С этим всё понятно.
>Итэр - уже пиздецки дорогой, но все еще экспериментальный в то время как нужен по крайней мере, ДЕМО.
Но без экспериментального не запилить даже демо, нужного уровня наработки технологий и знаний просто нет.

>Отсутствие жабы - как химическая ракета, шумно пыльно но быстро. Вот проекты новых чехлов на кикстартере и плодятся.
Профит от науки как плазменный двигатель, отличные характеристики, но работают только в сферическом политическом вакууме, которого просто нет.

Наука не существует "сама по себе" в сферическом вакууме. И в первую очередь исследуются те направления, которые могут принести профит в ближайшем будущем.
Сейчас столкнулись с одним парадоксом: интеллектуальных и экономических мощностей отдельно взятой страны не хватает для создания очередного проекта. Политического вакуума-тоже нет. Поэтому приходится сцепить зубы и выкручиваться. Другого просто не дано.

Аноним Суб 10 Май 2014 17:45:26 #427 №111216 

>>111184
>Наука не существует "сама по себе" в сферическом вакууме.
Ну, да об том и речь.

А ведро крабов - это когда есть ведро и есть крабы. Один краб из ведра выбраться может, но два или больше - нет. Один цепляется за край и пытается выкарабкаться. Но другие цепляются за него, и первый падает обратно.

Аноним Суб 10 Май 2014 19:44:49 #428 №111230 

>>111158 >>111169 А что-то более экзотичное типа эллипсоидов, пирамид, параллелепипедов, конусов? *эдрыяяяяя, призмы. И ещё более экзотичное типа тессерактов

Аноним Суб 10 Май 2014 20:33:24 #429 №111233 

>>110759
Гугли "звёздная машина" или "двигатель Шкадова". Но джетом будет долговато в любом случае всё это реализовывать. Если быстрее, то можно гигантский пузырь Алькубьерре создать вокруг планетной системы, тогда будем мы двигать её вместе с пространством, а не толкать одну лишь звезду. Ну естественно это сейчас звучит еще более фантастично. Нам этот пузырёк хоть бы вокруг маленькой кабины создать.

Аноним Суб 10 Май 2014 20:36:41 #430 №111234 

>>111233
> а не толкать одну лишь звезду
Сразу поясню местным занудам, что я не имел в виду, что толкая звезду, только она лишь и будет двигаться, а не планеты. Я вас знаю, вам лишь бы придраться и закидать другого чем нибудь нехорошим.

Аноним Суб 10 Май 2014 21:42:38 #431 №111240 

>>111233 Ты хотел сказать "Пузырь Алькуберке"?

Аноним Суб 10 Май 2014 22:08:22 #432 №111245 

>>111240
Ах, ну да, точно, Альбукерке. Спасибо, анон.

Аноним Суб 10 Май 2014 22:23:48 #433 №111250 
1399746228563.jpg

1) Нас защищает от космической радиации магнитное поле Земли, так? Почему тогда когда речь заходит о защиты космонавтов, то рассуждают о защите как о твёрдой преграде? Почему нельзя создать вокруг корабля сильное магнитное поле?
2) Вот есть термоядерный синтез. Атомы соединяются, дефект массы, все дела, тыры-пыры, выделяется энергия. Вот есть атомный распад. Атомы распадаются, высвобождается часть энергии, сдерживающей протоны и нейтроны вместе. Получается, что атомы распадаются - выделяется энергия, атомы синтезируются - снова выделяется энергия. В чём прикол?

Аноним Суб 10 Май 2014 22:40:11 #434 №111253 

>>111250
>В чём прикол?
Прикол в железе. Делишь железо - энергия поглощается, Прибавляешь к железу что угодно - энергия поглощается.

Аноним Суб 10 Май 2014 22:51:11 #435 №111255 

>>111250
>Нас защищает от космической радиации магнитное поле Земли, так?

Нет, не так. магнитное поле З. лишь отклоняет заряженные частицы к полюсам, и то, только низкоэнергичные. От космических лучей и солнечных вспышек направленных в сторону З. защищает атмосфера З. и результатами являются широкие атмосферные ливни и полярные сияния соответственно.

Не даром жи космонавтов проживших на орбита больше года учонные изучают под микроскопом.

Аноним Суб 10 Май 2014 23:27:05 #436 №111259 
1399750025122.png

Еще вопросы по поводу магнитного поля Земли.
1) Вот выходит из пикрелейтеда, что на где-то полюсах линии магнитной индукции должны быть направленны почти-что нормально поверхности Земли и входить в неё. При этом магнитный поток, как я понял, должен быть относительно велик. Вспоминая школьную физику, сразу приходит мысль о том, что если мы двигаем проводник перпендикулярно маг-ому полю, то на заряженные частицы проводника действует сила со стороны поля, из-за чего возникает ток. Почему тогда полярники спокойно ходят по полюсам, берут пробы и втыкают флажки, при этом их не шиндарахает током? Почему с помощью этого поля не индуцируют ток для обеспечения энергией полярных станций?
2) Почему в планетарных системах решающую роль на движение тел системы играет гравитация, а магнитное взаимодействие тел с магнитными полями не учитывается? Вот у Луны же внутренности в основном железные? Почему тогда не происходит поляризации магнитных доменов?
3) Откуда на всё это берётся энергия? Магнитное поле, как я понял, создаётся движущимся жидким металлическим ядром, у которого, из-за больших температур, много свободных зарядов. Так? Но откуда берётся тепло? Вот у звезды всё понятно - под большим давлением атомы сталкиваются, преодолевают Кулоновский барьер и происходит термоядерный синтез. Но ведь в центре Земли его нет. Чем же тогда объясняется? Большое давление ведь будет в таком случае говорить о том, что атомы будут близко друг к другу расположены. Откуда энергия?

Аноним Суб 10 Май 2014 23:33:53 #437 №111262 

>>111253
Не понял. А гелий? Гелий синтезировал - выделил энергию. Гелий расщепил - что будет?
>>111255
То есть основную радиационную опасность в космосе представляет собой гамма-излучение?

Аноним Суб 10 Май 2014 23:41:53 #438 №111263 

>>111259
>Чем же тогда объясняется?
Причин как обычно в таких темных делах, несколько. Относительно низкая теплопроводность астеносферы и коры, расщепление делящихся элементов и гравитационное перемещение внутреннего ядра луняшкой. Что-то народе магнитной мешалки.

Аноним Суб 10 Май 2014 23:48:28 #439 №111264 
1399751308259.gif

1) Сильно ли будет отличаться ход времени у наблюдателя на поверхности Земли от хода времени у наблюдателя, находящегося в центре большого войда? Ведь ход времени зависит от искривления пространства-времени, которое обеспечивает масса. Продолжая мысль, будет ли сильно отличаться ход времени в галактике, превышающую нашу в 2-3 раза? Ну и будут ли заметные изменения при приближении к центру галактики?
2) Почему скопление галактик представляют из себя галактические нити, а не что-то напоминающее по форме сами галактики, только состоящие не из звезд, а из галактик?
3) Смотрю на панорамы Млечного Пути, и он там изгибается аркой. Почему так?

Аноним Суб 10 Май 2014 23:51:50 #440 №111266 

>>111263
Блин, но у Венеры же нет крупных спутников, а её вон как колбасит, похуже Земляшки.

Аноним Суб 10 Май 2014 23:56:25 #441 №111270 

>>111262
>Гелий расщепил - что будет?
Будет поглощение. Это вам не вечный двигатель.
>представляет собой гамма-излучение?
Гамма конечно представляет некоторую опасность, оно вредно для здоровья, но нет, это не основная опасность.
Просто для того чтобы защитить космонавтов, магнитного поля недостаточно. Нужен экран, сопоставимый по защите с атмосферой. Свинец оче тяжелый плюс дает вторичку, воды надо тоже оче много, хотя проекты есть. Лучше всего подошел бы полиэтилен, но его тоже нужно немало. Так что скорее всего, комические корабли для многолетних перелетов будут иметь какую-то йобакомбинированную биологическую защиту, с магнитными отклонителями солнечных вспышек и синцово-полиэтиленными поглощаторами. Вопрос только как долго и как счастливо наши будущие кербонавты смогут жить в условиях сильных магнитных полей.

Аноним Вск 11 Май 2014 00:01:33 #442 №111272 

>>111266
Тяжелых элементов в ней больше, а магнитное поле - слабже.

Аноним Вск 11 Май 2014 00:03:44 #443 №111273 

>>111264
>панорамы Млечного Пути
>изгибается аркой.
Это баг фича всех панорам и широкоугольных объективов. Называется дисторсия.

Аноним Вск 11 Май 2014 00:09:16 #444 №111275 

>>111272
Магнитное поле - слабже, но ведь на ней постоянные извержения. Разве при нехватке энергии внутри планеты такие извержения были бы возможны? И что, что больше тяжелых элементов? Больше энергии от распада?
>>111270
Ясно, спасибо.

Аноним Вск 11 Май 2014 00:11:23 #445 №111276 

>>111264
>Сильно ли
Нет, не сильно, земляшка довольно легкая. Разница порядка нескольких триллионных долей секунды на секунду.

Аноним Вск 11 Май 2014 00:13:40 #446 №111277 

>>111276
Так вокруг Земляшки еще же целый Млечный Путь.

Аноним Вск 11 Май 2014 00:18:20 #447 №111278 

>>111275
>Больше энергии от распада?
Это.

Плюс ко всему, Венера ближе к печке. Вроде бы ерунда, и удельное остывание не сильно отличается от земного, но за 4.5 миллиарда лет разница в пришедшей солнечной радиации вполне ощутима.

Аноним Вск 11 Май 2014 00:27:23 #448 №111280 

>>111277
В этом деле важна напряженность гравитационного поля ускорению свободного падения, если чо. По сравнению с земляшкой, вклад млечки - ничтожен. И вообще млечный путь слишком размазан по пространству, можно сказать, это видимое нихуя.

Аноним Пнд 12 Май 2014 03:48:56 #449 №111355 

У Солнца есть атмосфера? Если нет то почему?

Аноним Пнд 12 Май 2014 04:21:49 #450 №111357 

>>111355
http://ru.wikipedia.org/wiki/%D0%A1%D0%BE%D0%BB%D0%BD%D1%86%D0%B5#.D0.90.D1.82.D0.BC.D0.BE.D1.81.D1.84.D0.B5.D1.80.D0.B0_.D0.A1.D0.BE.D0.BB.D0.BD.D1.86.D0.B0

Аноним Пнд 12 Май 2014 12:49:25 #451 №111365 

>>111275
Извержения не постоянные. Несколько сотен миллионов лет копится энергия от распада, после этого кору Венеры прорывает сразу во многих местах и на некоторое время воцаряется феерический пиздец.

Аноним Пнд 12 Май 2014 16:09:10 #452 №111375 

Анон, а что происходит со стартовыми ускорителями на ракетах? У них есть какие-то авто-парашюты или что-то в этом роде? Или они просто падают в рандомное место, недалеко от полигона, где собсно ракета и взлетела?

Аноним Пнд 12 Май 2014 19:20:29 #453 №111398 

>>111375
>просто падают в рандомное место, недалеко от полигона, где собсно ракета и взлетела

Аноним Пнд 12 Май 2014 19:31:34 #454 №111399 

>>111375
У шаттлов приводнялись на парашютах (твердотопливные ускорители). У Маска планируется контролируемое возвращение первой ступени. В остальных случаях >>111398, только место далеко не рандомное, всё-таки это не ведро картошки.

Аноним Втр 13 Май 2014 00:55:57 #455 №111464 

Спейсаны, онлаен с МКС: http://www.ustream.tv/channel/17074538

Можно ли в оп-пост треда этот стрим запилить?

Новый тред открывать постеснялся, т.к. я не могу, а этот-то всё стерпит.

Аноним Втр 13 Май 2014 07:01:09 #456 №111473 

>>111375
Падают в безлюдное место - тайгу, несудоходный район океана, и т.п. Не совсем рандомное, есть некий сектор падения.

Аноним Срд 14 Май 2014 13:07:11 #457 №111573 

Что будет с пилотируемой космонавтикой после того как МКС утопят? Есть в мире какие нибудь реально осуществляемые проекты, или пока только влажные фантазии типа роскосмосовских или mars one?

Аноним Срд 14 Май 2014 13:28:12 #458 №111574 

>>111573
Orion. В сентябре сего года полетит беспилотный прототип.

Аноним Срд 14 Май 2014 15:41:55 #459 №111580 

>>111574
А как же Драгоняша?
>>111573
МКС еще когда утопят. К этому времени новые пилотируемые корабли чуть ли не у всех будут.

Аноним Срд 14 Май 2014 15:57:58 #460 №111582 

>>111574
>>111580
А смысл. Есть в Орионах с драконами тренажеры, научные модули, тувалет наконец? Или долговременных орбитальных станций не планирует никто кроме кетая?

Аноним Чтв 15 Май 2014 00:12:22 #461 №111619 

>>111580
> А как же Драгоняша?
Он в первой версии, насколько я помню, не рассчитан на полеты дальше НОО.
>>111582
Наса планирует орион+надувной модуль а-ля Бигелоу. А может и от них же, не помню.

Аноним Чтв 15 Май 2014 10:36:38 #462 №111635 

>>111582
>тренажеры
>тувалет
Я честно говоря вообще не понимаю, зачем до сих пор летают в невесомости. Ну ладно МКС - тяжелое наследие военнизированного СССР, ладно у Роскосмоса старческая импотенция и из всех экспериментов они способны только богомольцев с иконами на пятидесятилетних Союзах запускать, но молодые перспективные могут же джва Дракона связать километровым тросом и раскрутить например?

Аноним Чтв 15 Май 2014 14:49:51 #463 №111653 

>>111635
Так основной смысл орбитальной лаборатории как раз в невесомости и заключается.

Аноним Чтв 15 Май 2014 15:28:30 #464 №111655 

>>111635
>джва Дракона связать километровым тросом и раскрутить например?
Это планируют применить при полёте на Марс чтобы за 1,5 года космонавты не разучились ходить.

Аноним Чтв 15 Май 2014 18:40:58 #465 №111661 

Спейсач, тут могут ответить на вопрос касательно геометрии на сфере?

Аноним Птн 16 Май 2014 05:01:28 #466 №111732 
1400202088952.png

>>108409
Чому не запускают ракеты со стратосферы?
Вес их при этом должен быть значительно меньше, как и расходы.

Аноним Птн 16 Май 2014 05:25:20 #467 №111735 

>>111732
Нет смысла. Попасть вверх - не особо большая проблема. Проблема разогнаться вокруг Земли, почти вся энергия ракеты уходит на это.

Аноним Птн 16 Май 2014 08:14:28 #468 №111738 
1400213668655.png

>>111732
Потому что орбита это не вот это...

Аноним Птн 16 Май 2014 08:15:06 #469 №111739 
1400213706345.png

>>111738
...орбита - это вот это.

Аноним Птн 16 Май 2014 08:16:05 #470 №111740 
1400213765505.png

>>111739
Не та пикча. (но тоже сойдет) Вот что такое орбита.

Аноним Птн 16 Май 2014 08:16:46 #471 №111741 
1400213806524.png

>>111740
Трудно не попасть в космос, трудно остаться в нём.

Аноним Птн 16 Май 2014 08:33:53 #472 №111742 
1400214833685.jpg

>>111739
>>111738
>>111735
Спасибо, добра.
Но ведь стартовать чуть повыше один хрен экономнее, будет больше топлива для разгона, не?

Аноним Птн 16 Май 2014 09:55:34 #473 №111746 

>>111742
Немного экономней, но не оправдывает строительства воздушного шара с гелием размером с город для поднятия многотонной йобы. Да и технологически намного сложней с него стартовать, и возможностей меньше. Не стоит того, короче. Даже воздушный старт с самолета и тот сомнительная затея.

Аноним Птн 16 Май 2014 10:16:44 #474 №111747 

>>111735
Какой процент топлива используется на преодоление силы тяжести и сопротивления атмосферы?

Аноним Птн 16 Май 2014 10:35:04 #475 №111748 
1400222104165.jpg

>>111747
Около 10% от общей характеристической скорости ракеты, если мы говорим про НОО и типичные РН. По массе получится больше, ибо формула Циолковского имеет экспоненциальный характер, но все равно это не стоит свеч для больших ракет на текущем этапе развития технологий.

Воздушный старт (и то с самолета, не с шара) имеет смысл в основном для маленьких ракет, сейчас из ракет-носителей так запускаются только Pegasus, способные нести всего лишь полтонны на орбиту. Раньше так запускались экспериментальные NOTS-EV-2, которые достигли орбиты лишь неофициально, согласно байкам.

Сейчас ещё разрабатывают Pegasus II, который будет нести уже порядочные 6 тоннна НОО или 2 тонны на геопереходную, может и выйдет чего путного.

Аноним Птн 16 Май 2014 10:38:17 #476 №111749 

>>111748
Олсо, надо помнить, что к точке запуска с шара ракета уже имеет нехилую скорость, а с шара она стартует с нуля. Так что там получится вовсе не 10% разницы в дельте, а гораздо меньше, 10% это общие гравитационные потери (от которых шар почти не избавляет). Да и траектория получится более крутой, что тоже может создать некоторые затруднения.

Аноним Птн 16 Май 2014 13:08:56 #477 №111761 

>>111748
А я думал, луна хороша как раз тем, что нет атмосферы и помжно почти сразу разгоняться на орбиту.
А ведь действительно, набрать первую космическую тот ещё пиздец.

Аноним Птн 16 Май 2014 13:17:55 #478 №111763 

>>111761
Тащемта она этим действительно хороша: например электромагнитный разгонный трек, который бесполезен на Земле, на Луне был бы вполне к месту.

Аноним Птн 16 Май 2014 15:03:46 #479 №111790 

>>111761
На Луне и разгоняться надо всего до 1.6 км/c.

Аноним Птн 16 Май 2014 22:08:51 #480 №111834 

Интенсивно ли сверхмассивные чёрные дуыры излучают гравитационные волны? Ну т.е. порядка светового излучения падающего вещества или больше/меньше?

Аноним Втр 20 Май 2014 11:23:22 #481 №112075 

Наибольшее количество созвездий видно зимой? Если ты в России наблюдаешь.

Аноним Втр 20 Май 2014 11:29:08 #482 №112077 

>>111834
Сверхмассивные черные дыры сами по себе не излучают гравитационные волны, для излучения гравитационных волн нужна динамическая система, например две ЧД (или например нейтронные звезды) в тесной двойной системе.

Аноним Втр 20 Май 2014 15:26:15 #483 №112087 

>>112075
Россия, знаешь ли очень большая.

Аноним Втр 20 Май 2014 18:58:09 #484 №112093 

Можно ли укрыться от гамма всплеска? В бункерах каких нибудь. И с какой скоростью летят частицы? Можно ли их сдетектить и передать сигнал на землю

Аноним Втр 20 Май 2014 21:31:46 #485 №112100 

>>112093
>>передать сигнал на землю
Сигнал тревоги придет вместе с гамма всплеском, что очень информативно.

Аноним Срд 21 Май 2014 09:56:58 #486 №112120 

>>112093
Ну я думаю донные бактерии, простейшие и, возможно, некоторые животные выживут в самой пучине окияна. Всё-таки такая толща воды хорошо поглощает даже гамму.

Аноним Срд 21 Май 2014 10:17:03 #487 №112123 

>>112087
Широта ДС2.

Аноним Срд 21 Май 2014 10:19:21 #488 №112124 

>>112120
> Всё-таки такая толща воды хорошо поглощает даже гамму.
Двачую, тут пацаны даже рассчитали безопасность купания в бассейне с отработанным ядерным топливом.
http://chtoes.li/page/spent-fuel-pool

Аноним Срд 21 Май 2014 14:05:11 #489 №112129 

>>112124
Это не пацаны, а перевод xkcd https://what-if.xkcd.com/29/

Аноним Срд 21 Май 2014 23:25:15 #490 №112136 

>>112123
На широте ДС2 летом вообще нихуя не видно.
Белые ночи-с, сударь.

Аноним Чтв 22 Май 2014 12:30:16 #491 №112158 

>>112136
Ну, в августе уже нормально, тащемта.

Аноним Чтв 22 Май 2014 21:27:54 #492 №112174 

>>108409
http://hi-news.ru/space/nasa-planiruet-privezti-obrazcy-s-marsa.html
Скажити пажалысто, это норм план или план уровня осовения Луны Роскосым?

Аноним Чтв 22 Май 2014 22:01:57 #493 №112175 
1400781717776.jpg

>>112174
Плана освоения Луны собственно нет в природе, пока есть только пиздеж одного политика. MSR это скорее нечто уровня Марс-Грунт, последователя запоротого Фобос-Грунта: мусолят его уже очень давно, просто иногда мусолят активней. Рано или поздно запустят, но пока только разговоры.
http://en.wikipedia.org/wiki/Mars_sample_return_mission

Аноним Птн 23 Май 2014 00:01:25 #494 №112176 

>>112174
>осовения Луны
Луна смотрю сегодня совсем осовела.
типа фикс

>>112175
Спасибо.

Аноним Птн 23 Май 2014 00:19:08 #495 №112180 

Почему МКС всегда повернута одним боком к поверхности планеты?

Аноним Птн 23 Май 2014 01:43:24 #496 №112182 

>>112180
У нее три режима ориентации, на самом деле. Нужный подбирается исходя из опасности столкновения с мусором (разворот помогает уменьшить площадь профиля), требуемой эффективности солнечных батарей (зависит от угла бета) и стыковки с кораблями.

Аноним Птн 23 Май 2014 13:14:31 #497 №112191 

А что находится за космосом?
Ведь сами подумайте, раньше люди думали, что земля плоская, можно упасть с края земли, потом узнали, что она круглая. Далее узнали о существовании космоса, об орбитах. Теперь мы исследуем марс, другие планеты, запускаем спутники. Как долго осталось ждать до того, чтобы узнать что же на самом деле находится за космосом?

sageАноним Птн 23 Май 2014 13:15:28 #498 №112192 

>>112191
>что же на самом деле находится за космосом?
Анус твоей мамаши, школотун.

Аноним Птн 23 Май 2014 13:19:24 #499 №112193 

>>112192
Ты остряк? По-моему ты не даёшь отчёт себе в том, что ты лишь набор хим процессов с животными инстинктами и ничего более. Ты появился из ничего и в ничто после смерти уйдёшь.

sageАноним Птн 23 Май 2014 13:23:10 #500 №112194 

>>112193
>смерти
Она прекрасна.
Засим общение завершаем, ибо норма зоофилии на сегодня перевыполнена.
/дискасс

Аноним Птн 23 Май 2014 13:25:26 #501 №112195 

>>112191
>А что находится за космосом?
http://elementy.ru/lib/430484

пока не бамплимит Аноним Птн 23 Май 2014 13:42:27 #502 №112196 
1400838147609.png

Если Луна обращена к Земле одной стороной, как получается, что космонавты снимали восходы и закаты Земли на горизонте Луны?

Есть ли на Луне точка, в которой Земля никогда не двигается на небе? Ну, если бы она постоянно была видима.

Аноним Птн 23 Май 2014 14:50:55 #503 №112202 

>>112196
>Если Луна обращена к Земле одной стороной, как получается, что космонавты снимали восходы и закаты Земли на горизонте Луны?
Они летали вокруг по орбите, не?

Есть ли на Луне точка, в которой Земля никогда не двигается на небе? Ну, если бы она постоянно была видима.
Везде.

Аноним Птн 23 Май 2014 19:50:27 #504 №112220 

>>112196
>космонавты снимали восходы и закаты Земли на горизонте Луны?
А точно снимали?

Аноним Птн 23 Май 2014 23:43:47 #505 №112234 

>>112196
Снимки, где Земля восходит над луной, были сделаны с командного модуля, то есть с орбиты, а там каждый час Земляшка "восходит".

Ну и вообще на ~20% лунной поверхности можно наблюдать восходы Земли раз в месяц.

Аноним Птн 23 Май 2014 23:47:05 #506 №112235 

>>112234
Решил приложить пикчу и чтобы вы думали?
>Этот файл был уже загружен здесь >>107593

Аноним Птн 13 Июн 2014 17:44:26 #507 №113871 

>>108409 Мудрецы космача наверняка не пожалеют своего свободного от передёргивания времени, чтобы объяснить анону почему вытесняющий газ хранят в шарообразных баках?

Аноним Втр 08 Июл 2014 12:04:15 #508 №116553 

>>113871
Наиболее прочная форма оболочки при заданной массе материала.

comments powered by Disqus

Отзывы и предложения